T/C II Exam 3 version 3

Réussis tes devoirs et examens dès maintenant avec Quizwiz!

What meds are used to ripen the cervix and dilate it?

Misoprostol or Prostaglandins

A nurse in a prenatal clinic is caring for client who asks what her estimated date of delivery will be if her last menstrual period was May 4, 2015. Which of the following is the appropriate response by the nurse? February 11, 2016 February 27, 2016 April 27, 2016 April 11, 2016

February 11, 2016.

A nurse is providing teaching about expected gestational changes to a client who is at 12 weeks of gestation. Which of the following statements by the client indicates a need for further teaching? "I will reduce my stress level." "I will tell my doctor before using home remedies for nausea." "I will monitor my weight gain during the remaining months." "I will use only nonprescription medications while pregnant."

I will use only nonprescription medications while pregnant.

risk factors for hypoglycemia in the newborn

Preterm infants Infant of diabetic mother (IDM) SGA infant The smaller of twins Mother preeclampsia Male babies

placental previa signs and symptoms

bright red painless vaginal bleeding

Blood infection in infants

sepsis neonatorum

The nurse is assessing a pregnant client at her 20-week visit. Which breast assessment should the nurse anticipate documenting? Enlarged lymph nodes Darkened breast areolae Deeply fissured nipples Slack, soft breast tissue

Darkened breast areolae Explanation: As part of the pigment changes that occur with pregnancy, breast areolae become darker. The breast tissue should not be softer or slacker than before. There should not yet be any lymph enlargement, and the nipples should not have fissures.

Bishop score

Determines maternal readiness for labor by evaluating whether the cervix is favorable by rating cervical dilation, effacement, consistency, position, and station High score of 13 and low of 0. Score of 8 or greater is desired for Induction of Labor

A nurse is assessing a newborn who has a coarctation of the aorta. Which of the following should the nurse recognize is a clinical manifestation of coartation of the aorta? Increased blood pressure in the arms with decreased blood pressure in the legs Decreased blood pressure in the arms with increased blood pressure in the legs Increased blood pressure in both the arms and the legs Decreased blood pressure in both the arms and the legs

Increased blood pressure in the arms with decreased blood pressure in the legs There is a narrowing next to the ductus arteriosus that results in an increased pressure proximal to the defect, with a decreased pressure distal to the obstruction. Therefore, an increased blood pressure in the arms with a decreased blood pressure in the legs would be a clinical manifestation of a coarctation of the aorta.

What is often the cause of preterm rupture of membranes?

Infection in the uterus

If a patient presents with bright red painless bleeding and states she has had no prenatal care, what do you NOT do?

Vaginal / cervical exam, because this could be a placenta previa and you could stick your fingers through the placenta and cause a huge problem, like more bleeding.

babies with _________________ have an increased risk for jaundice

cephalohematoma

Risk factors for Cervical insufficiency

cervical trauma, D&C

Which finding is most worrisome in a client in her 26th week of pregnancy? nosebleeds generalized hair loss facial edema a hyperpigmented rash over the maxillary region bilaterally

facial edema Explanation: Generalized hair loss, hyperpigmented maxillary rash (chloasma), and nosebleeds are usually benign and common in pregnancy. Facial edema after the 24th week of gestation may indicate gestational hypertension.

What is a positive sign of pregnancy?

fetal movement felt by examiner

Nursing interventions for TTN

Administer oxygen, IV fluid and electrolytes, abstain from oral feedings, pulse ox monitoring, other respiratory assessments and interventions

Nuchal cord

Cord around the baby's neck:

How to test for congenital cardiac defect

Pulse ox on right arm and right leg The baby passes the screening if pre-and postductal O2 saturation are less than 3% difference

Placental abruption risks

Significant maternal and fetal morbidity and mortality; can lead to DIC

What condition of the newborn includes the failure to clear long fluid, mucus, debris

Transient tachypnea of the newborn

A woman experiences an amniotic fluid embolism as the placenta is delivered. The nurse's first action would be to: administer oxygen by mask. increase her intravenous fluid infusion rate. tell the woman to take short, catchy breaths. put firm pressure on the fundus of her uterus.

administer oxygen by mask. Explanation: An amniotic embolism quickly becomes a pulmonary embolism. The woman needs oxygen to compensate for the sudden blockage of blood flow through her lungs.

placenta previa delivery method

c section

Cervical insufficiency

passive and painless dilation of the cervix leading to recurrent preterm births during the second trimester in the absence of other causes

Abnormal implantation of the placenta over or near the cervical OS

placental previa

Neonatal risks of preterm rupture of membranes

respiratory distress syndrome, sepsis, intraventricular hemorrhage, and death.

In preparing for a preconception class, the nurse plans to include a discussion of potential risk factors. Which risk factor would be most important to include? the importance of healthy lifestyle importance of taking adequate vitamin and mineral supplements family history of pregnancy complications the use of OTC drugs with teratogens

the use of OTC drugs with teratogens Explanation: Risk factors for adverse pregnancy have been demonstrated by statistics gathered for smoking during pregnancy, consuming alcohol during pregnancy, not taking adequate folic acid supplements during pregnancy, being obese, taking prescription or OTC drugs that are known teratogens, and having a preexisting condition that can negatively affect pregnancy if unmanaged.

cerclage

used for incompetent cervixes. suturing of the cervix to prevent it from dilating prematurely during pregnancy, thus decreasing the chance of a spontaneous abortion

What is a common cause of intrauterine growth restriction

smoking

Placenta previa risk factors

previous previa, scar tissue, AMA, multiple gestation, smoking

what gestation determines full term labor?

39 wks, 0 days - 40 weeks, 6 days

A nurse in a prenatal clinic is reviewing the health record of a client who is at 28 weeks of gestation. The history includes one pregnancy, terminated by elective abortion at 9 weeks; the birth of twins at 36 weeks; and a spontaneous abortion at 15 weeks. According to the GTPAL system, which of the following describes the client's current status? 4-0-1-2-2 3-0-2-0-2 2-0-0-2-0 4-2-0-2-2

4-0-1-2-2

What gestation determines late term labor?

41 wks, 0 days - 6 days

A nurse is assessing a newborn who has developmental dysplasia of the hip (DDH). Which of the following findings should the nurse expect? Absent plantar reflexes Lengthened thigh on the affected side Inwardly turned foot on the affected side Asymmetric thigh folds

Asymmetric thigh folds. Gluteal and thigh skin folds that are not equal and symmetric is a sign of DDH.

A nurse in a provider's office is collecting a health history from a client who is at risk for primary osteoporosis. Which of the following findings is a risk factor for the development of osteoporosis? A. Obesity B. Sedentary lifestyle C. Long-term use of diuretics D. Prolonged stress

B. Sedentary lifestyle A sedentary lifestyle places the client at risk for osteoporosis. Regular, weight-bearing exercises help to build bone tissue.

what indicates that interventions for hypoglycemia in the newborn are working?

BG level above 45

A nurse is speaking with a 35-year-old client who has fibrocystic disease of the breasts. At which of the following times should the nurse inform the client that manifestations are the most evident? Before menstruation begins After menstruation ends During cold weather During hot weather

Before menstruation begins Manifestations of benign fibrocystic breast changes include painful breasts, smooth moveable lumps, and possible swelling of the breasts, which tends to worsen premenstrually. Reducing salt and caffeine intake sometimes helps.

Risk factors for TTN

C-section, macrosomia, LGA

A nurse is caring for a newborn who has hydrocephalus. Which of the following manifestations should the nurse expect to find? Over-riding suture lines Dilated scalp veins Hypertension A backward sloping appearance of the forehead.

Dilated scalp veins Manifestations of hydrocephalus in newborns include dilated scalp veins, separated sutures, and, in late infancy, frontal enlargement.

A nurse is caring for a client who has preeclampsia and is being treated with magneisum sulfate IV. The client's respiratory rate is 10/min and deep-tendon reflexes are absent. Which of the following actions should the nurse take? Discontinue the medication infusion. Prepare for an emergency cesarean birth. Assess maternal blood glucose. Place the client in Trendelenburg position.

Discontinue the medication infusion.

A nurse is caring for a client who is in premature labor and is receiving terbutaline. The nurse should monitor the client for which of the following adverse effects that should be reported to the provider? Headaches Nervousness Tremors Dyspnea

Dyspnea

A nurse is caring for a client who is scheduled for a maternal serum alpha-fetoprotein test at 15 weeks of gestation. The nurse provides which of the following explanations about this test to the client? This test assesses fetal lung maturity. It assesses various markers of fetal well-being. This test identifies an Rh incompatibility between the mother and fetus. It is a screening test for spinal defects in the fetus.

It is a screening test for spinal defects in the fetus.

hypoglycemia signs and symptoms in the newborn

Lethargy, sleepiness, limpness Poor feeding and sucking, vomiting Hypothermia, temperature instability Pallor, cyanosis Apnea, irregular respirations, respiratory distress, tachypnea Tremors, jitteriness, seizure activity, irritability, eye rolling High-pitched cry Exaggerated Moro reflex *jitteriness, poor feeding, vomiting, irritability

A nurse is intitiating the newborn's plan of care. Complete the following sentence by using the list of options. The nurse should first address the client's ____________ followed by the client's ___________________.

Respiratory status Temperature

Major complication of LGA baby

Shoulder dystocia and birth trauma

Premature Rupture of Membranes (PROM)

a rupture (breaking open) of the membranes (amniotic sac) before labor begins (one hour or more)

Subchorionic hemorrhage

bleeding behind the placenta that can sometimes cause miscarriages

What anatomic area should be examined when assessing Montgomery glands (Montgomery tubercles)? breasts perineum abdomen thorax

breasts Explanation: Montgomery glands (Montgomery tubercles) are sebaceous glands on the areola of the breasts and are prominent during pregnancy.

Nuchal cord treatment

can be manually reduced or cut on perineum

placental abruption signs and symptoms

painful dark red vaginal bleeding

A urinalysis is done on a client in her third trimester. Which result would be considered abnormal? 2+ Protein in urine Specific gravity of 1.010 Trace of glucose Straw-like color

2+ Protein in urine Explanation: During pregnancy, there may be a slight amount of glucose found in the urine due to the fact that the kidney tubules are not able to absorb as much glucose as there were before pregnancy. However, there should be minimal protein in the urine. A specific gravity of 1.010 and a straw- like color are both normal findings.

What gestation determines post-term labor?

42 wks, 0 days and beyond

symptoms of placenta previa

Painless Bright red vaginal bleeding Soft uterus Normal fetal hr

what are the risk factors to having an at risk newborn?

- low socioeconomic status - limited/no prenatal care - exposure to environmental dangers - pre-existing maternal conditions (heart disease, HTN, diabetes, thyroid issues, renal disease) - pregnancy complications (placental abruption, oligohydramnios, preterm labor, PROM, preeclampsia)

A woman's prepregnant weight is within the normal range. During her second trimester, the nurse would determine that the woman is gaining the appropriate amount of weight when her weight increases by which amount per week? 2/3 lb (0.30 kg) 1 lb (0.45 kg) 2 lb (0.90 kg) 1.5 lb (0.68 kg)

1 lb (0.45 kg) Explanation: The recommended weight gain pattern for a woman whose prepregnant weight is within the normal range would be 1 lb (0.45 kg) per week during the second and third trimesters. Underweight women should gain slightly more than 1 lb (0.45 kg) per week. Overweight women should gain about 2/3 lb (0.30 kg) per week.

3 things magnesium sulfate does for the pregnant woman

1. prevent seizures (lowers seizure threshold) 2. protect baby's brain at time of delivery 3. slows down labor process

A nurse on an obstetrics-gynecology unit is planning care for four clients after receiving change of shift report. Which of the following clients should the nurse assess first? A client who is a 1 day postpartum after a late term miscarriage A client who had a bilateral tubal ligation 12 hr previously A client who is 4 days postpartum and has mastitis A client admitted 1 hr ago for an ectopic pregnancy

A client admitted 1 hr ago for an ectopic pregnancy A client who has an ectopic pregnancy is unstable. The client is at risk for rupture of the fallopian tube, hemorrhage, and shock. Nursing care requires frequent monitoring every 15 min, IV access for fluid resuscitation. The client may also require blood transfusions, oxygen, and pain management. Therefore this client is the highest priority.

An RN from the maternal-newborn unit is being floated to a medical surgical unit. Which of the following clients should the charge nurse plan to assign to RN? A client who has terminal end-stage renal disease A client who has acute pancreatitis A client who is one-day postoperative following a total abdominal hysterectomy A client who had a stroke and is to be admitted

A client who is one-day postoperative following a total abdominal hysterectomy The nurse who floats to another unit must have the skills to provide safe care to clients. This client is stable. This is an appropriate assignment for the RN.

A nurse is reviewing the health history of a client who has a new prescription for a combined oral contractive (COC). The nurse recognizes that which of the following client medications can interfere with the effectiveness of the COC? Antihypertensives Anticonvulsants Antioxidants Antiemetics

Anticonvulsants Anticonvulsants when taken simultaneously with COCs can decrease their effectiveness. The anticonvulsants included are: phenytoin, phenobarbital, carbamazepine, oxcarbazepine, topiramate, and primidone.

A nurse is caring for a client who has urinary incontinence. Which of the following actions should the nurse implement to prevent the development of skin breakdown? Apply a moisture barrier ointment to the client's skin. Clean the client's skin and perineum with hot water after each episode of incontinence. Check the client's skin every 8 hr for signs of breakdown. Request a prescription for the insertion of an indwelling urinary catheter.

Apply a moisture barrier ointment to the client's skin. Skin that remains in contact with urine for prolonged periods is at risk for maceration and breakdown. After cleansing and drying the client's skin, the nurse should apply a moisture barrier ointment to prevent further contact of the skin with urine.

A nurse is caring for a client who is beginning to breastfeed her newborn after delivery. The new mother states, "I don't want to take anything for pain because I am breastfeeding." Which of the following statements should the nurse make? A. "You need to take pain medications so you are more comfortable." B. "We can time your pain medication so that you have an hour or two before the next feeding." C. "All medications are found in breast milk to some extent." D. "You have the option of not taking pain medication if you are concerned."

B. "We can time your pain medication so that you have an hour or two before the next feeding." This answer provides the client an option that allows for administration of pain medication but minimizes the effect it will have on the newborn while breastfeeding.

A nurse is caring for a client who asks to be screened for cervical cancer because a relative has been diagnosed with it. Which of the following tests should the nurse expect the provider to use? A. A serum prolactin level B. A Papanicolaou test C. A vaginal ultrasound D. An endometrial biopsy

B. A Papanicolaou test A Papanicolaou test involves sampling cells from the cervix to detect abnormal cells and growth. The nurse should recommend the client have an annual Pap test between ages 21 to 29, and every 5 years from ages 30 to 65.

A nurse on the obstetric unit is caring for a client who experienced abruptio placentae. The nurse observes petechiae and bleeding around the IV access site. The nurse should recognize that this client is at risk for which of the following complications? A. Anaphylactoid syndrome of pregnancy B. Disseminated intravascular coagulation C. Preeclampsia D. Puerperal infection

B. Disseminated intravascular coagulation Clinical manifestations of disseminated intravascular coagulation (DIC) include oozing from intravenous access and venipuncture sites; petechiae, especially under the site of the blood pressure cuff; spontaneous bleeding from the gums and nose; other signs of bruising; and hematuria.

A nurse is reinforcing teaching about newborn care with a postpartum client. Which of the following statements by the client indicates a need for further teaching? "I will use mild soap." "I will use a basin during bathing." "Baby powder will help prevent a diaper rash." "I will test the water on my wrist for temperature before bathing."

Baby powder will help prevent a diaper rash. Lotions, creams, oils, or powders can alter a newborn's skin and provide a medium for bacterial growth or cause an allergic response. Powders can be inhaled, leading to respiratory distress. This statement requires the nurse to clarify instruction on newborn care.

What can help with fetal lung development in preterm labor?

Betamethasone steroid

A nurse is reviewing a newborn's laboratory results. Which of the following findings is the nurse's priority? Platelets 200,000/mm3 Bilirubin 19 mg/dL Blood glucose 45 mg/dL Hemoglobin 22 g/dL

Bilirubin 19mg/dL Bilirubin 19 mg/dL is above the expected reference range for a newborn at 4 hr of age. A bilirubin level greater than 15 mg/dL or an increase by more than 6 mg/dL in 24 hr is pathologic or nonphysiologic jaundice. Pathologic jaundice is a result of an underlying disease and occurs before 24 hr of age; therefore, this is the nurse's priority finding.

A pregnant client in her third trimester, lying supine on the examination table, suddenly grows very short of breath and dizzy. Concerned, she asks the nurse what is happening. Which response should the nurse prioritize? The uterus requires more blood in a supine position. Blood is trapped in the vena cava in a supine position. Cerebral arteries are growing congested with blood. Sympathetic nerve responses cause dyspnea when a woman lies supine.

Blood is trapped in the vena cava in a supine position. Explanation: Supine hypotension syndrome, or an interference with blood return to the heart, occurs when the weight of the fetus rests on the vena cava. Cerebral arteries should not be affected. Mean arterial pressure is high enough to maintain perfusion of the uterus in any orientation. The sympathetic nervous system will not be affected by the supine position.

Which information provided by a client would be considered a presumptive sign of pregnancy? Reports of increased hunger Breast tenderness Weight gain Ballottement

Breast tenderness Explanation: Presumptive signs of pregnancy are things reported by the woman to the health care provider and occur early in pregnancy. Breast tenderness is a common sign reported by women in early pregnancy but is not a definitive sign. Reports of increased hunger and weight gain could be caused by any disorder or could be normal responses to eating cycles. Ballottement occurs late in the pregnancy and is a probable sign.

A nurse is planning care for a preterm newborn. Which of the following nursing interventions to promote development should be included in the plan of care? A. Position the newborn to promote extension of muscles B. Use fingertips when calming newborn C. Cluster the newborn's care activities D. Keep the newborn I a well-lit nursery

Cluster the newborn's care activities. By clustering activities and organizing care, the nurse prevents excessive interruptions and allows the newborn extended periods of rest and energy conservation that promote development.

A nurse is caring for a client who is HIV positive and is one day postoperative following an appendectomy. The nurse should wear a gown as PPE when taking which of the following actions? Talking to the client at the bedside Administering an intermittent IV bolus medication Completing a dressing change Administering an IM injection

Completing a dressing change Standard precautions require personal protective equipment when there is a risk of contact with body fluids. A dressing change does present a risk for coming into contact with body fluids.

A nurse is admitting a client who has severe preeclampsia at 35 weeks of gestation and is reviewing the provider's orders. Which of the following orders requires clarification? Assess deep tendon reflexes every hour. Obtain a daily weight. Continuous fetal monitoring Ambulate twice daily.

D. Ambulate twice daily A provider's order to allow the client to ambulate requires clarification. The client who has severe preeclampsia should be placed on bedrest in a quiet, nonstimulating environment to prevent seizures and promote optimal placental blood flow.

A nurse is assessing a newborn who was born at 42.5 weeks of gestation. Which of the following findings should the nurse expect? A. Copious vernix B. Scant scalp hair C. Increased subcutaneous fat D. Dry, cracked skin

D. Dry, cracked skin A newborn who is postmature has dry, cracked skin.

A nurse is caring for an adolescent client who is gravida 1 and para 0. The client was admitted to the hospital at 38 weeks of gestation with a diagnosis of preeclampsia. Which of the following findings should the nurse identify as inconsistent with preeclampsia? 1+ pitting sacral edema 3+ protein in the urine Blood pressure 148/98 mm Hg Deep tendon reflexes of +1

Deep tendon reflexes of +1

Characteristics of post term newborn

Dry cracked skin without vernix or lanugo Long fingernails Profuse scalp hair Long thin body with loose skin and nonexistent fat layers Meconium staining

A nurse is interviewing a client who has a possible diagnosis of endometriosis. Which of the following findings in the client's history should the nurse recognize as consistent with a diagnosis of endometriosis? A history of pelvic inflammatory disease (PID). Abdominal bloating starting several days before menses. An atypical Papanicolaou smear at her last clinic visit. Dysmenorrhea that is unresponsive to NSAIDs.

Dysmenorrhea that is unresponsive to NSAIDs. Endometriosis is a condition in which the type of tissue that lines the uterus implants in locations outside the uterus. This typically causes pelvic pain around the time of the menstrual period but can cause pain at other times in the cycle. THe discomfort is often unrelieved by the use of NSAIDs.

A nurse is assessing a pregnant client. The nurse understands that hormonal changes occur during pregnancy. Which hormones would the nurse most likely identify as being inhibited during the pregnancy? FSH and LH LH and MSH FSH and T4 T4 and GH

FSH and LH Explanation: During pregnancy, FSH and LH are both inhibited as there is no need to develop a follicle and release an ovum. There is an increase in the secretion of T4 and MSH. There is a decrease in the production of GH and MSH but not an inhibition.

A nurse is completing an assessment of a 1 month old newborn. Which of of the following developmental skills is an expected finding? Displays a social smile Follows movements of objects with eyes Reacts to sounds by turning head Makes babbling sounds

Follows movements of objects with eyes A 1-month-old infant is able to follow movements with their eyes.

Which information is most important in order to decrease the risk of complications if the client decides to work until her due date? Frequent rest periods Adequate sleep Eat light meals Flat shoes

Frequent rest periods Explanation: It is common to have a client work until she goes into labor as long as she has had a low-risk pregnancy. Frequent rest periods are stressed, if possible, as the client progresses throughout the work day. The other options are good suggestions for any client at the end of pregnancy. Reference:

A pregnant woman comes to the clinic for a prenatal visit for her third pregnancy. She reveals she had a previous miscarriage at 12 weeks and her 3-year-old son was born at 32 weeks. How should the nurse document this woman's obstetric history? G3, T1, P0, A2, L1 G2, T1, P2, A1, L2 G2, T0, P1, A1, L1 G3, T0, P1, A1, L1

G3, T0, P1, A1, L1 Explanation: The woman's obstetric history would be documented as G3, T0, P1, A1, L1. G (gravida) = 3 (past and current pregnancy), T (term pregnancies) = 0, P (number of preterm pregnancies) = 1, A (number of pregnancies ending before 20 weeks viability to include miscarriage) = 1, and L (number of living children) = 1.

A nurse is completing a health history for a client who is at 6 weeks of gestation. The client informs the nurse that she smokes one pack of cigarattes per day. The nurse should advise the client that smoking places the client's newborn at risk for which of the following complications? Hearing loss Intrauterine growth restriction Type 1 diabetes mellitus Congenital heart defects

Intrauterine growth restriction

What do you not want to happen with a placenta previa?

Labor, because it tears the placenta, so we do not want to allow any signs of labor to progress

important consideration for placenta previa

NO CERVICAL CHECKS OR VAGINAL EXAMS - can poke through the placenta

signs and symptoms of meconium aspiration

Pallor cyanosis/apnea Slow heart rate Low apgars (below 6) Respiratory distress Barrel shaped chest Decreased air movement rales and rhonchi *yellowish/pale green skin and nails

A nurse is caring for a client who is in labor and has an epidural anesthesia block. The client's blood pressure is 80/40 mmHg and the fetal heart rate is 140/min. Which of the following is the priority nursing action? Elevate the client's legs. Monitor vital signs every 5 min. Notify the provider. Place the client in a lateral position

Place client in a lateral position.

A nurse is planning care for a newborn who has spina bifida. Which of the following actions should be included in the plan of care? Obtain rectal temperatures. Place the newborn in the prone position. Cover the lesion with a dry dressing. Apply snug, clean diapers.

Place the newborn in the prone position. Placing the newborn in the prone position prevents trauma to the lesion. The newborn's knees should be assessed for evidence of skin breakdown.

A nurse caring for a client who has severe preeclampsia and is receiving magnesium sulfate IV at 2g/hr. Which of the following findings indicates that it is safe for the nurse to continue the infusion? Diminished deep-tendon reflexes Respiratory rate of 16/min Urine output of 50 mL in 4hr Heart rate of 56/min

Respiratory rate of 16/min

A nurse is teaching self-management to a client who has hepatitis B. Which of the following instructions should the nurse include in the teaching? You may donate blood 6 months after completing the medication regimen. Consume a high-protein diet. Rest frequently throughout the day. Take acetaminophen every 4 hr, as needed, for discomfort

Rest frequently throughout the day. Limiting activity is usually recommended until the symptoms of hepatitis have subsided. The nurse should recommend the client rest frequently throughout the day to reduce the metabolic demands upon the liver and decrease energy demands.

A pregnant client at 24 weeks' gestation calls the clinic crying after a prenatal visit, where she had a pelvic exam. She states that she noticed blood on the tissue when she wiped after voiding. What initial statement by the nurse would explain this finding? It is possible she is losing her mucus plug, which can cause bloody show. She may have a bleeding disorder so she needs to come back to the clinic for blood work. Some bleeding during pregnancy is not uncommon and this finding is expected. The cervix is very vascular during pregnancy, so spotting after a pelvic exam is not unusual.

The cervix is very vascular during pregnancy, so spotting after a pelvic exam is not unusual. Explanation: Slight bleeding after a pelvic exam in a pregnant woman is common due to the vascularity of her cervix during pregnancy. Suggesting a bleeding disorder is frightening and not substantiated by the data. Bleeding is not a normal finding during pregnancy and losing the mucus plug occurs at the end of pregnancy, just prior to labor.

A nurse is providing teaching about Kegal exercises to a group of clients who are in the third trimester of pregnancy. Which of the following statements by a client indicates understanding of the teaching? These exercises help prevent constipation. These exercises help pelvic muscles to stretch during birth. They can help reduce back aches. They can prevent further stretch marks.

These exercises help pelvic muscles to stretch during birth. Kegel exercises improve the strength of perineal muscles, facilitating stretching and contracting during childbirth.

A nurse in a prenatal clinic is caring for a client who believes that she might be pregnant because she feels the baby moving. Which of the following statements should the nurse make? "This is a presumptive sign of pregnancy." "This is a probable sign of pregnancy." "This is a possible sign of pregnancy." "This is a positive sign of pregnancy."

This is a presumptive sign of pregnancy.

A nurse is caring for a new mother who is concerned that her newborn's eyes cross. Which of the following statement is a therapeutic response by the nurse? "I will call your primary care provider to report your concerns." "I will take your baby to the nursery for further examination." "This occurs because newborns lack muscle control to regulate eye movement." "This is a concern, but strabismus is easily treated with patching."

This occurs because newborns lack muscle control to regulate eye movement This addresses the client's concerns because it provides information that addresses her concerns. The eyes of newborns are structurally incomplete and muscle control is not fully developed for 3 months.

A nurse is caring for a client who is to start chemotherapy for advanced breast cancer. She tells the nurse she is worried about the adverse effects of the treatment. Which of the following responses should the nurse make? "I will have your provider discuss the adverse effects with you before the treatment begins." "Someone from the American Cancer Society will be here soon to answer your questions." "What is it about the adverse effects that concern you?" "I agree. Sometimes the adverse effects can be worse than the disease."

What is it about the adverse effects that concern you? With this response, the nurse takes responsibility for answering the client's concerns rather than passing them to someone else. It also exemplifies the therapeutic communication technique of exploring, as it invites the client to share her concerns.

Placenta previa

abnormal implantation over or near the cervical OS Complete or partial, marginal or low-lying

The nurse plays a major role in assessing the progress of labor. The nurse integrates understanding of the typical rule for monitoring labor progress. Which finding would the nurse correlate with this rule? fetus descends 1 cm per hour cervix dilates 1 cm per hour cervix dilates 2 cm per hour fetus descends 2 cm per hour

cervix dilates 1 cm per hour Explanation: A simple rule for evaluating the progress of labor is expecting 1 cm per hour of cervical dilation (dilatation). If the cervix fails to respond to uterine contractions by dilating and effacing, then dysfunctional labor must be ruled out.

A pregnant client states she was unable to breastfeed her last child because her breasts did not produce milk. She desires to breastfeed this child. Which hormones would the nurse monitor to during this pregnancy? progesterone and relaxin oxytocin and progesterone estrogen and human placental lactogen (hPL) relaxin and human chorionic gonadotropin (hCG)

estrogen and human placental lactogen (hPL) Explanation: Estrogen aids in developing the ductal system of the breasts in preparation for lactation during pregnancy. hPL prepares the mammary glands for lactation. Progesterone supports the endometrium of the uterus to provide an environment conducive to fetal survival. Oxytocin is responsible for uterine contractions, both before and after birth. Oxytocin is also responsible for milk ejection during breastfeeding.

postterm pregnancy

gestation of the fetus that extends beyond 42 weeks (294 days since last menstrual period)

The nurse is preparing to teach a community class to a group of first-time parents. Which information should the nurse include concerning what the pregnant woman's partner may experience as a normal response? feeling distanced from the mother no changes, only the mother has changes during pregnancy desire to be the woman and give birth physical symptoms similar to the mother

physical symptoms similar to the mother Explanation: Couvade syndrome is the occurrence of physical symptoms by the partner, similar to the physical symptoms of the mother. Other emotional symptoms may occur, but they are typically on a person-to-person basis.

main difference between placentral abruption and placenta previa

placental abruption: abdominal pain with bleeding, uterine tenderness, firm uterus placental previa: no pain at all, bleeding, soft uterus, normal FHR

What is the Coombs test?

screening tool for Rh incompatibility

A client who has just given a blood sample for pregnancy testing in the health care provider's office asks the nurse what method of confirming pregnancy is the most accurate. The nurse explains the difference between presumptive symptoms, probable signs, and positive signs. What should the nurse mention as an example of a positive sign, which may be used to diagnose pregnancy? laboratory test of a urine specimen for hCG laboratory test of a blood serum specimen for hCG visualization of the fetus by ultrasound absence of a period

visualization of the fetus by ultrasound Explanation: There are only three documented or positive signs of pregnancy: 1) demonstration of a fetal heart separate from the mother's, 2) fetal movements felt by an examiner, and 3) visualization of the fetus by ultrasound. The absence of a period is an example of a presumptive symptom, which is a symptom that, when taken as a single entity, could easily indicate other conditions. Laboratory tests of either urine or blood serum for human chorionic gonadotropin (hCG) are examples of probable signs of pregnancy, which are objective and so can be verified by an examiner.

Target O2 saturation for one minute, two minutes, three minutes, four minutes, five minutes, 10 minutes

1 min - 60-65% 2 min - 65-70% 3 min - 70-75% 4 min - 75-80% 5 min 80-85% 10 min 85-95%

The nurse is assessing a pregnant client at 20 weeks' gestation and obtains a hemoglobin level. Which result would be a cause for concern? 12.8 g/dl 11.9 g/dl 10.6 g/dl 11.2 g/dl

10.6 g/dl Explanation: The average hemoglobin level at term is 12.5 g/dl. The hemoglobin level is considered normal until it falls below 11 g/dl.

The nurse assesses a 20-week gestational client at a routine prenatal visit. What will the nurse predict the fundal height to be on this client experiencing an uneventful pregnancy? 16 cm 12 cm 20 cm 24 cm

20 cm Explanation: Between weeks 18 and 32 the fundal height in centimeters should match the gestational age of the pregnancy. At 20 weeks' the fundal height should be at the umbilicus. A fundal height smaller than expected can indicate that the original dates were miscalculated, oligohydramnios, or that the fetus is smaller than expected. If the fundal height is larger than expected this can indicate multiple gestation, the original dates were miscalculated, polyhydramnios, or a molar pregnancy.

The nurse is caring for a client after experiencing a placental abruption (abruptio placentae). Which finding is the priority to report to the health care provider? hematocrit of 36% (0.36) 45 ml urine output in 2 hours platelet count of 150,000 mm3 hemoglobin of 13 g/dl (130 g/L)

45 ml urine output in 2 hours Explanation: The nurse knows a placental abruption places the client at high risk of hemorrhage. A decreased urine output indicates decreased perfusion from blood loss. The hematocrit, hemoglobin, and platelet counts are all within expected levels.

Before becoming pregnant, a woman's heart rate averaged 72 beats per minute. The woman is now 15 weeks' pregnant. The nurse would expect this woman's heart rate to be approximately: 95 beats per minute. 85 beats per minute. 100 beats per minute. 90 beats per minute.

85 beats per minute. Explanation: During pregnancy, heart rate increases by 10 to 15 beats per minute between 14 and 20 weeks of gestation, and this elevation persists to term. Therefore, a prepregnancy heart rate of 72 would increase by 10 to 15 beats per minute to a rate of 82 to 87 beats per minute.

Rubella antibodies IgG: 8 international units/mL (antibody index 0.8) The nurse is reviewing rubella antibody testing results (above) for a pregnant client at 8 weeks' gestation. What action does the nurse anticipate based on these results? Administer the measles-mumps-rubella (MMR) vaccine in the third trimester. Administer the measles-mumps-rubella (MMR) vaccine as soon as possible. Administer the measles-mumps-rubella (MMR) vaccine after weaning. Administer the measles-mumps-rubella (MMR) vaccine postpartum.

Administer the measles-mumps-rubella (MMR) vaccine postpartum. Explanation: A rubella IgG antibody index of 0.7 to 1.0 (7 to 10 international units/ml) is equivocal and an additional dose of measles-mumps-rubella (MMR) vaccine is indicated in order to develop sufficient immunity to rubella. The MMR vaccine is a live vaccine and cannot be given in pregnancy. It should be administered postpartum. Breastfeeding is not a contraindication to live vaccine administration; the MMR vaccination does not need to be deferred until after weaning.

A pregnant mother may experience constipation and the increased pressure in the veins below the uterus can lead to development of what problem? Umbilical hernia Varicose veins Hemorrhoids Gastrointestinal reflux

Hemorrhoids Explanation: The displacement of the intestines and possible slowed motility of the intestines can lead to constipation in the pregnant woman. This, along with elevated venous pressure, can lead to development of hemorrhoids.

What to monitor for with post term newborns

Hypothermia, hypoglycemia, polycythemia, cold stress, seizures, meconium aspiration, respiratory distress syndrome

A nurse is caring for a newborn who has myelomeningocele. Which of the following nursing goals has the priority in the care of this infant? A. educate the parents about the defect B. provide age-appropriate stimulation C. promote maternal-infant bonding D. maintain the integrity of the sac

Maintain the integrity of the sac. Myelomeningocele is a congenital disorder that causes the spine and spinal canal to not close prior to birth, which results in the spinal cord, meninges, and nerve roots protruding out of the child's back in a fluid-filled sac. Before surgery, the infant must be handled carefully to reduce damage to the exposed spinal cord and to maintain the integrity of the sac.

A nurse is admitting a client who is at 36 weeks gestation and has painless, bright red vaginal bleeding. The nurse should recognize this finding as an indication of which of the following conditions? Abruptio placentae Placenta previa Precipitous labor Threatened abortion

Placenta previa

A client is receiving treatment for stage IV ovarian cancer and asks the nurse to discuss her prognosis. The client plans to have aggressive surgical, radiation, and chemotherapy treatments. Which of the following prognoses should the nurse discuss with the client? Good Guarded Poor Very good

Poor At this advanced stage, the prognosis for ovarian cancer is poor. Ovarian cancer is the leading cause of death from female reproductive cancers. Survival rates are low because it is not often discovered until its late stages.

symptoms of placental abruption

Sudden onset of intense LOCALIZED PAIN with dark red vaginal bleeding, uterine tenderness and firmness, hypertonic UC's, hypovolemic shock

A nurse is reviewing risk factors for osteoporosis with a group of nursing students. The nurse should include that which of the following types of medication therapy is a risk factor for osteoporosis? Thyroid hormones Anticoagulants NSAIDs Cardiac glycosides

Thyroid hormones Long-term use of a synthetic thyroid hormone, such as levothyroxine, can accelerate bone loss.

what diseases are the neonate protected against with erythromycin antibiotic ointment applied to the eyes at birth?

chlamydia and gonorrhea

Leading cause of preterm labor

cigarette smoking

most PROMINENT cause of preterm labor

cigarette smoking

A nurse is instructing a woman who is contemplating pregnancy about nutritional needs. To reduce the risk of giving birth to a newborn who has a neural tube defect, which of the following information should the nurse include in the teaching? Limit alcohol consumption. Increase intake of iron-rich foods. Consume foods fortified with folic acid. Avoid foods containing aspartame.

Consume foods fortified with folic acid. Increased consumption of folic acid in the 3 months prior to conception, as well as throughout the pregnancy, reduces the incidence of neural tube defects in the developing fetus.

A nurse is preparing to assess a newborn who is postmature. Which of the following findings should the nurse expect? SATA Cracked, peeling skin Positive Moro reflex Short, soft fingernails Abundant lanugo Vernix in the folds and creases

Cracked, peeling skin positive Moro reflex Cracked, peeling skin is correct. Physical findings that indicate postmaturity in a newborn (gestational age of greater than 42 weeks) include cracked, peeling skin .Positive Moro reflex is correct. Reflexes that are present in a postmature newborn are the same as those that are present in a mature newborn. These reflexes include a positive Moro reflex.

The client states that the first day of her last menstrual period is March 23. The nurse is most correct to calculate using Naegele rule that the estimated date of delivery is: December 30 January 30 December 16 November 23

December 30 Explanation: Using Naegele rule, since the first day of the client's last menstrual period is March 23, 7 days are added leading to the 30th. Subtracting 3 months from March is December. Thus, December 30 is the estimated date of delivery.

A client's membranes rupture. The nurse observes the fetal heart rate drop from 156 to 110. The nurse inspects the client's perineum and sees a loop of umbilical cord. What is the nurse's priority concern in this situation? Increased risk for infection Decreased fetal oxygenation Increased risk for placental abruption Decreased strength of uterine contractions

Decreased fetal oxygenation Explanation: When there is a cord prolapse the cord becomes compressed, blood flow is interrupted, and there is decreased oxygen available to the fetus resulting in fetal distress. There is a slight increased risk for postbirth infection, but it is not the priority at this time. A cord prolapse does not increase the risk for placental abruption nor does it decrease the strength of uterine contractions.

A graduate nurse (GN) is caring for a client being induced via oxytocin infusion. The client is currently reporting a headache and is vomiting. The graduate nurse thinks that the client is getting near the end of labor. However, the GNs preceptor intervenes by performing which interventions immediately after hearing this report? Select all that apply. administering IV ondansetron for the nausea/vomiting discontinuing the oxytocin infusion increasing IV fluid rate notifying the health care provider immediately calling respiratory therapy to obtain ABGs on this client

Discontinuing the oxytocin infusion notifying the health care provider immediately Explanation: A second side effect of oxytocin is that it can result in decreased urine flow, possibly leading to water intoxication. This is first manifested by a headache and vomiting. If the nurse observes these danger signs in a client during induction of labor, the client should report them immediately and halt the infusion. Ondansetron may be appropriate but is not the priority. The headache and vomiting are due to water intoxication, so fluids should be decreased not increased. At this point, ABGs are not the priority intervention.

A nurse is completing a health hx for a client who is at 6 weeks gestation. The client informs the nurse that she smokes one pack of cigarettes per day. The nurse should advice the client that smoking places the clients newborn at risk for which of the following complications? Hearing loss Intrauterine growth restriction Type 1 diabetes mellitus Congenital heart defects

Intrauterine growth restriction. Clients who smoke place their newborns and themselves at risk for diverse complications, including fetal intrauterine growth restriction, placental abruption, placenta previa, preterm delivery, and fetal death.

A client who is uncertain when her LMP occurred is given an EDD of April 23 after the first ultrasound. Based on this information, the nurse determines the client's LMP was probably which day? July 13 July 21 July 19 July 16

July 16 Explanation: According to Naegele rule, the last menstrual period was July 16th. Take the LMP and add 7 days and subtract 3 months; if finding the LMP from the EDD, subtract 7 days and add 3 months.

A client calls to cancel an appointment for the first prenatal visit after reporting a home pregnancy test is negative. Which instruction should the nurse prioritize? Wait until after two missed menstrual periods. Use a diluted urine specimen. Keep the appointment. Refrain from eating for 4 hours before testing.

Keep the appointment. Explanation: Although home pregnancy tests are accurate 95% of the time, they may still have false positives or false negatives, and the client needs to seek prenatal care and confirmation from her health care provider. Diluting the urine, waiting to miss a second period, or eating before the test would have no effect. The tests look for hCG, which is not affected.

A nurse is assessing an older adult client who has osteoporosis. Which of the following spinal deformities should the nurse expect to find in this client? Lordosis Ankylosis Kyphosis Scoliosis

Kyphosis Kyphosis, a forward "stooping" posture with a loss of height, is an angulation of the posterior curve of the thoracic spine, usually a result of osteoporosis. It is most common in older adults and increases with aging and vertebral fractures.

The nurse is caring for a client suspected to have a uterine rupture. The nurse predicts the fetal monitor will exhibit which pattern if this is true? Late decelerations Variable decelerations Early decelerations Mild decelerations

Late decelerations Explanation: When the fetus is being deprived of oxygen the fetus will demonstrate late decelerations on the fetal monitoring strip. This is an indication the mother is in need of further assessment. Early decelerations are a normal finding. Variable decelerations usually coincide with cord compression.

A nurse is completing the admission assessment of a newborn. Which of the following anatomical landmarks should the nurse use when measuring the newborn's chest circumference? Sternal notch Nipple line Xiphoid process Fifth intercostal space

Nipple Line Rationale: The nurse should measure the newborn's chest circumference at the nipple line.

Immediately after giving birth to a full-term infant, a client develops dyspnea and cyanosis. Her blood pressure decreases to 60/40 mm Hg, and she becomes unresponsive. What does the nurse suspect is happening with this client? aspiration congestive heart failure placental separation amniotic fluid embolism

amniotic fluid embolism Explanation: With amniotic fluid embolism, symptoms may occur suddenly during or immediately after labor. The woman usually develops symptoms of acute respiratory distress, cyanosis, and hypotension.

The nurse is assessing a client who believes she is pregnant. The nurse points out a more definitive assessment is necessary due to which sign being considered a probable sign of pregnancy? Fatigue Positive home pregnancy test Nausea and vomiting Amenorrhea

Positive home pregnancy test Explanation: A urine pregnancy test is considered a probable sign of pregnancy as the hCG may be from another source other than pregnancy. Fatigue, amenorrhea, and vomiting are presumptive or possible signs of pregnancy and can also have other causes.

A nurse in a clinic is teaching information about cervical polyps with a client who has a new diagnosis. Which of the following information should the nurse include in the teaching? Avoid using tampons during menstruation. Cervical polyps are a precursor to the development of cervical cancer. Cervical polyps affect women before the age of 40. Postcoital bleeding may occur.

Postcoital bleeding may occur. The client may experience postcoital bleeding, because the polyps are soft, fragile, and bleed when touched.

Differentiate between primary Apnea and secondary apnea

Primary apnea - decrease or lack of respiration and decreasing heart rate that will improve with tactile stimulation Secondary apnea - occurs with a fall and heart rate and blood pressure. Stimulation will not reverse and assisted ventilation must be provided

Risks of preterm rupture of membranes

Risks to mom include intra-amniotic infection, postpartum infection, endometritis, and death.

what is important for newborns receiving phototherapy

cover their eyes and genitals turn every 2 hours remove all clothing except the diaper

Some pregnant women hire a trained professional to provide support during pregnancy and birth, to provide emotional support during labor and birth, and to aid in establishing breastfeeding. What is the name of the woman who takes this role? partera midwife doula pregnancy aide

doula Explanation: The pregnant woman may hire a doula to provide support for labor and birth and help with establishing breastfeeding. A doula can also provide support for the postpartum period.

how to prevent hypoglycemia in the newborn

early feedings

A nurse who has been caring for a pregnant client understands that the client has pica and has been regularly consuming soil. For which condition should the nurse monitor the client? constipation iron-deficiency anemia tooth fracture inefficient protein metabolism

iron-deficiency anemia Explanation: Pica is characterized by a craving for substances that have no nutritional value. Consumption of these substances can be dangerous to the client and her developing fetus. The nurse should monitor the client for iron-deficiency anemia as a manifestation of the client's compulsion to consume soil. Consumption of ice due to pica is likely to lead to tooth fractures. The nurse should monitor for inefficient protein metabolism if the client has been consuming laundry starch as a result of pica. The nurse should monitor for constipation in the client if she has been consuming clay.

hypoglycemia in newborn blood glucose level

less than 40-45

Premature separation of the placenta from the uterus

placental abruption

A nurse is caring for a newborn and observes signs of diaphoresis, jitteriness, and lethargy. Which of the following actions should the nurse take? A. Obtain blood glucose by heel stick. B. Initiate phototherapy. C. Monitor the newborn's blood pressure. D. Place the newborn in a radiant warmer.

A. Obtain blood glucose by heel stick. The newborn is exhibiting early signs of hypoglycemia. The nurse should obtain blood by heel stick to check glucose. A therapeutic serum glucose level for a newborn is 40 to 60 mg/dL. Less than 40 mg/dL indicates hypoglycemia. Other findings of hypoglycemia include poor feeding, tremors, hypothermia, flaccid muscle tone, irregular respirations, apnea, cyanosis, and a weak, shrill cry. Early breastfeeding also should be encouraged to prevent hypoglycemia.

A nurse is providing teaching to the mother of a newborn born small for gestational age. Which of the following should the nurse include as a possible cause of this condition? A. Placental insufficiency B. Preterm delivery C. Fetal hyperinsulinemia D. Perinatal asphyxia

A. Placental insufficiency Placental insufficiency is a cause of small for gestational age. It can result from maternal infections, embryonic placental deficiency, teratogens, or chromosomal abnormalities.

What should you do when resuscitating a baby if meconium is absent or present

Absent - check to see if the baby is vigorous (has a heart rate over 100 bpm, good muscle tone, making respiratory efforts) Present - clear the babies mouth and nose and dry the baby, simulate, reposition

The nurse is planning to contact the provider regarding the newborn's status. Which of the following prescriptions should the nurse anticipate? Select 3. Swaddle the newborn. Encourage the birthing parent to breastfeed. Continue NAS scoring as prescribed. Administer naloxone for NAS scores greater than 24. Administer oral morphine.

Administer oral morphine. Swaddle the newborn. Continue NAS scoring as prescribed. Administer oral morphine is correct. The nurse should administer oral morphine to assist with decreasing the withdrawal findings in the newborn. The dosage of the medication is adjusted based on the NAS scores of the newborn. Encourage the birthing parent to breastfeed is incorrect. The nurse should not allow the client to breastfeed because of their heroin use. However, if the client was prescribed methadone the nurse should encourage the client to breastfeed on demand because the newborn is experiencing neonatal abstinence syndrome (NAS). Swaddle the newborn is correct. The nurse should swaddle the newborn and reduce the environmental stimuli for newborns with NAS to assist with decreasing the manifestations the newborn is experiencing. Administer naloxone for NAS scores greater than 24 is incorrect. The nurse should not administer naloxone to a newborn who has a diagnosis of NAS. Naloxone is contraindicated in newborns born with opioid toxicity and can cause severe manifestations including seizures. Continue NAS scoring as prescribed is correct. The nurse should continue conducting NAS scoring as prescribed in order to evaluate the newborn's clinical findings and status. The score obtained will assist with determining the dosage of morphine to administer.

A woman tells the nurse that she is going to use a home pregnancy test to determine whether she is pregnant. Which precautions should the nurse give her? Refrain from eating for 4 hours before testing. Arrange for prenatal care if the test is positive. Use a diluted urine specimen. Wait until after two missed menstrual periods.

Arrange for prenatal care if the test is positive. Explanation: Home pregnancy testing can be accurate as soon as a period is missed; it should not take the place of prenatal care.

A nurse is caring for a client who is 1 day postoperative following gynecologic surgery and reports incisional pain. Which of the following actions should the nurse take first? Determine the time the client last received pain medication. Measure the client's vital signs, including temperature. Ask the client to rate her pain on a scale from 0 to 10. Reposition the client and offer her a back rub.

Ask the client to rate her pain on a scale from 0 to 10. Using evidence-based practice, the nurse should first determine the severity of the client's pain by using a standard pain scale. Then the nurse can plan the appropriate interventions.

A nurse is caring for a client who has HIV. Which of the following laboratory values is the nruse's priority? Positive Western blot test CD4-T-cell count 180 cells/mm3 Platelets 150,000/mm3 WBC 5,000/mm3

CD4-T-cell count 180 cells/mm3 A CD4-T- cell count less than 180 cells/mm indicates that the client is severely immunocompromised and is at high risk for infection. Therefore, this value is a priority for the nurse to report to the provider.

Interventions for Prolapsed Cord

Call for help immediately !!! Notify provider Apply pressure with 2 fingers to the fetal presenting part to elevate it off the cord Reposition pt to Trendelenburg or knee-chest Administer high-flow oxygen Wrap exposed cord with sterile saline-soaked gauze *** Prepare for immediate birth (likely by C-section)

Cord compression

Can reduce blood flow from the placenta to the fetus Causes: abnormal presentation, inadequate pelvis, presenting part at a high station, multiple gestations, prematurity, PROM, and Polyhydramnios Can cause fetal asphyxia

causes of possterm pregnancy

Cause is largely unknown, genetics, obesity, primiparity

Before calling the health care provider to report a slow progression or an arrest of labor, several assessments need to be made. What other maternal assessment does the nurse need to make prior to calling the health care provider? Make sure the epidural medication is turned down. Assess vital signs every 30 minutes. Make sure the client is lying on her left side. Check for a full bladder.

Check for a full bladder. Explanation: A full bladder can interfere with the progress of labor, so the nurse must be sure that the client has emptied her bladder.

A nurse is assessing an older adult client who reports a sudden onset of urinary incontinence. The nurse should recognize which of the following conditions can cause incontinence in the older adult client? Nephrosclerosis Uremia Diverticulitis Cystitis

Cystitis A sudden onset of urinary incontinence or increased confusion can indicate the presence of a urinary tract infection or bacterial cystitis in the older adult client.

tachysystole labor pattern

-uterine hyperstimulation: greater than 5 contractions per 10 minutes -this is bad and it is dysfunctional

A nurse is teaching a class about preventative care to client who are at risk for acquiring viral hepatitis. Which of the following information should the nurse include in the presentation? Avoid covering sores with bandages. Avoid handwashing after eating. Avoid foods prepared with tap water. Avoid eating meat.

Avoid foods prepared with tap water. To decrease the risk for acquiring viral hepatitis, clients should prepare foods with purified water.

A nurse is caring for a newborn 4 hr after birth. Which of the following actions should the nurse include in the plan of care to prevent jaundice? A. Begin phototherapy B. Initiate early feeding C. Suction excess mucus with bulb syringe D. Prepare for an exchange blood transfusion

B. Initiate early feeding Prevention of jaundice can be facilitated best by early and frequent feeding, which stimulates intestinal activity and passage of meconium. Jaundice occurs due to elevated serum bilirubin, which is excreted primarily in the newborn's stool. Physiologic jaundice manifests after 24 hr and is considered benign. However, bilirubin may accumulate to hazardous levels and lead to a pathologic condition.

A nurse is preparing to administer an injection of Rh(D) immunoglobulin. The nurse should understand that the purpose of this injection is to prevent which of the following newborn complications? Hydrops fetalis Hypobilirubinemia Biliary atresia ?Transient clotting difficulties

Hydrops fetalis

A nurse is caring for a term macrosomic newborn whose mother has poorly controlled type 2 diabetes. The newborn has respiratory distress syndrome. The nurse should be aware that the most likely cause of respiratory distress is which of the following? Hyperinsulinemia Increased deposits of fat in the chest and shoulder area Brachial plexus injury Increased blood viscosity

Hyperinsulinemia High levels of maternal glucose increase the production of fetal insulin. High fetal insulin levels interfere with the production of surfactant.

A nurse in a provider's office is caring for a client who is at 34 weeks of gestation and at risk for placental abruption. The nurse should recognize that which of the following is the most common risk factor for abruption? Cocaine use Hypertension Blunt force trauma Cigarette smoking

Hypertension

A nurse is teaching a client about preventing osteoporosis. Which of the following statements by the client indicates a need for further teaching? "I will reduce my intake of sodium." "I will decrease my intake of caffeine." "I will limit my intake of soft drinks." "I will reduce my intake of vitamin K-rich foods."

I will reduce my intake of vitamin K-rich foods. Vitamin K is necessary for bone health. The nurse should instruct the client to increase her intake of vitamin K-rich foods—such as green, leafy vegetables—to promote bone health.

During an exam, the nurse notes that the blood pressure of a client at 22 weeks' gestation is lower, and her heart rate is 12 beats per minute higher than at her last visit. How should the nurse interpret these findings? Combined, both of these findings are very concerning and warrant further investigation. Both findings are normal at this point of the pregnancy. The blood pressure should be higher since the cardiac volume is increased. The heart rate increase may indicate that the client is experiencing cardiac overload.

Both findings are normal at this point of the pregnancy. Explanation: A pregnant woman will normally experience a decrease in her blood pressure during the second trimester. An increase in the heart rate of 10 to 15 beats per minute on average is also normal, due to the increased blood volume and increased workload of other organ systems. Hormonal changes cause the blood vessels to dilate, leading to a lowering of blood pressure.

Which information provided by a client would be considered a presumptive sign of pregnancy? breast tenderness

Breast tenderness Explanation: Presumptive signs of pregnancy are things reported by the woman to the health care provider and occur early in pregnancy. Breast tenderness is a common sign reported by women in early pregnancy but is not a definitive sign. Reports of increased hunger and weight gain could be caused by any disorder or could be normal responses to eating cycles. Ballottement occurs late in the pregnancy and is a probable sign.

A nurse is caring for a middle adult female client who reports that her menstrual periods have become irregular and she has been having hot flashes. The nurse should expect the client to have which of the following manifestations associated with early menopause? Urinary retention Decreased blood pressure Dryness with intercourse Elevation in body temperature above 37.8° C (100° F)

Dryness with intercourse Menopause, the cessation of a woman's menstrual periods, occurs when the ovaries stop making estrogen. Because of the changes in the vagina, some women can have dryness, discomfort, or pain during sexual intercourse.

The nurse cares for a pregnant client at the first prenatal visit and reviews expected changes that will occur during pregnancy. Which information will the nurse include in the education? Blood pressure decreases in the third trimester. During pregnancy blood volume can increase by at least 40%. Hemoglobin levels rise significantly during pregnancy. Pregnancy typically causes a decrease in respiratory rate.

During pregnancy blood volume can increase by at least 40%. Explanation: The pregnant woman can experience a blood volume increase by approximately 40% to 50% above prepregnancy levels by the end of the third trimester. Pregnancy results in an increased respiratory rate to provide oxygen to both the mother and fetus. Hemoglobin levels are usually low during pregnancy because of hemodilution of red blood cells, which is termed physiologic anemia of pregnancy. Blood pressure usually reaches a low point mid-pregnancy and, thereafter, increases to prepregnancy levels by the third trimester.

The nurse cares for a pregnant client at the first prenatal visit and reviews expected changes that will occur during pregnancy. Which information will the nurse include in the education? Pregnancy typically causes a decrease in respiratory rate. During pregnancy blood volume can increase by at least 40%. Hemoglobin levels rise significantly during pregnancy. Blood pressure decreases in the third trimester.

During pregnancy blood volume can increase by at least 40%. Explanation: The pregnant woman can experience a blood volume increase by approximately 40% to 50% above prepregnancy levels by the end of the third trimester. Pregnancy results in an increased respiratory rate to provide oxygen to both the mother and fetus. Hemoglobin levels are usually low during pregnancy because of hemodilution of red blood cells, which is termed physiologic anemia of pregnancy. Blood pressure usually reaches a low point mid-pregnancy and, thereafter, increases to prepregnancy levels by the third trimester.

The perinatal educator is instructing on various emotions commonly experienced during labor. Which complication of anxiety is most important to stress? Shortness of breath Gestational hypertension Fetal tachycardia Dystocia

Dystocia Explanation: Many women experience an array of emotions during labor, which may include fear, anxiety, helplessness, desire to be alone, and weariness. These emotions can lead to psychological stress, which indirectly can cause dystocia. Dystocia is a prolonged labor as the tense woman is fighting against the labor process. Shortness of breath may occur with a panic attack. Gestational hypertension occurs during pregnancy. Fetal tachycardia is not commonly associated with maternal anxiety.

At her prenatal visit a client reports that she cannot find any shoes that are comfortable. Assessment of her legs reveals dependent edema. The nurse suggests that the client attempt which actions to help reduce the edema? Select all that apply. Drink 6 to 8 glasses of water each day. Wear knee-high support stockings. Elevate feet and legs when sitting or lying. Avoid foods high in sodium, sugar, and fats. When lying down, lie on the right side.

Elevate feet and legs when sitting or lying. Avoid foods high in sodium, sugar, and fats. Drink 6 to 8 glasses of water each day. Explanation: Dependent edema is usually the result of pressure put on the veins preventing adequate blood flow to return to the heart. Appropriate suggestions to reduce dependent edema include elevating feet and legs when sitting or lying down; avoiding foods that are high in sodium, sugar, and fats; drinking at least 6 to 8 glasses of water per day; avoid wearing knee-high stockings; and lying on the left side to keep the gravid uterus off the vena cava to return blood to the heart.

The nurse has determined that based on the client's physical examination she is at high risk for developing varicose veins. Which suggestions might the nurse teach the client to help reduce her risk? Select all that apply. Elevate the feet and legs. Sit in a hot tub at least three times a week. Use knee-high support hose. Walk daily. Use thigh-high support hose.

Elevate the feet and legs Walk daily Use thigh-high support hose Vascular changes during pregnancy manifested in the integumentary system include varicosities of the legs, vulva, and perineum. Varicose veins commonly are the result of distention, instability, and poor circulation. Various interventions to reduce the risk of developing varicosities include elevating both legs when sitting or lying down; avoiding prolonged standing or sitting; walking daily for exercise; avoiding tight clothing or knee-high hosiery; and wearing support hose if varicosities are a preexisting condition to pregnancy.

A nurse is talking with a young adult client who has a family history of osteoporosis. Which health promotion activity should the nurse recommend as a possible preventive measure? Increase sodium intake. Have a bone-density scan each year. Engage in weight-bearing exercise regularly. Drink a cup of coffee each morning.

Engage in weight-bearing exercise regularly. Regular weight-bearing exercise, such as walking and stair-climbing, increases bone density and can reduce the risk for osteoporosis.

A mother comes in with her 17-year-old daughter to find out why she has not had a menstrual cycle for a few months. Examination confirms the daughter is pregnant with a fundal height of approximately 24 cm. The nurse interprets this finding as indicating that the daughter is approximately how many weeks pregnant? 18 22 24 20

24 Explanation: By 20 weeks' gestation, the fundus of the uterus is at the level of the umbilicus and measures 20 cm. A monthly measurement of the height of the top of the uterus in centimeters, which corresponds to the number of gestational weeks, is commonly used to date the pregnancy. Therefore for this client, the additional 4 cm would be the equivalent of 4 additional weeks making the gestational age of 24 weeks.

The nurse is reviewing client data following a regular monthly appointment at 6 months' gestation. Which fundal height requires no further intervention? 24 cm 32 cm 30 cm 18 cm

24 cm Explanation: An anticipated fundal height for 24 weeks' gestation (6 months) is 24 cm. Between 18 and 32 weeks' gestation, the fundal height in centimeters should match the gestational age. All of the other measurements would require further intervention.

when is a TC bilirubin test done on a newborn

24 hours of age

A nurse is providing nutritional teaching to a client who has osteoporosis. Which of the following foods should the nurse recommend as being the highest in calcium? 1 cup carrot strips 3 oz canned salmon 1 cup chopped chicken breast 1 plain baked potato

3 oz canned salmon The nurse should recommend canned salmon as a food to increase calcium intake. A 3 oz serving of canned salmon contains 197 mg of calcium.

A 22-year-old client comes to the walk-in clinic complaining of fatigue, breast heaviness and extreme tenderness, and a clear vaginal discharge. What question would the nurse ask this client? Have you been sexually active in the past 2 months? Do you have vaginal itching? Do you have a family history of breast cancer? Are you taking oral contraceptives?

Have you been sexually active in the past 2 months? Explanation: The client is presenting with presumptive or subjective symptoms of pregnancy. Given her symptoms and age, asking about sexual activity is the most appropriate question. Whether she is taking an oral contraceptive will not assist in identifying the cause of her symptoms. If she has vaginal itching, the underlying cause of her symptoms needs to be identified before treatment can be prescribed. Asking about family history is part of a comprehensive health history, but is not the priority based on the client's presentation.

A woman who is 4 months pregnant notices frequent heart palpitations and leg cramps. She is anxious to learn how to alleviate these. Which nursing diagnosis would best apply to her? Risk for ineffective breathing pattern related to pressure of the growing uterus Impaired urinary elimination related to inability to excrete creatine from her muscles Health-seeking behaviors related to ways to relieve discomforts of pregnancy Pain related to severe complications of pregnancy

Health-seeking behaviors related to ways to relieve discomforts of pregnancy Explanation: Health-seeking behaviors is a diagnosis used to describe clients who are actively interested in learning ways to improve their health.

To prepare his 4-year-old son for a new baby, a father should use which statement? "Mother will need to spend a lot of time with the new baby." "A new baby will make our family bigger but not change our love for you." "The new baby will need your bed so we're buying you a new one." "It will be fun to have a sister or brother to give your old toys to."

"A new baby will make our family bigger but not change our love for you." Explanation: It is important that siblings see a new family member as adding to the family (not displacing them) to prevent jealousy.

The nurse provides education to a postterm pregnant client. What information will the nurse include to assist in early identification of potential problems? "Be sure to measure 24-hour urine output daily." "Monitor your bowel movements for constipation." "Increase your fluid intake to prevent dehydration." "Continue to monitor fetal movements daily."

"Continue to monitor fetal movements daily." Explanation: The nurse will teach the postterm client to monitor fetal movements (kick counts) daily to help determine if the fetus is experiencing distress. A 24-hour urine is needed for postterm clients; however, this is not collected daily. Although all pregnant clients should avoid dehydration, there is no indication this client needs to increase her fluid intake and this will not help identify potential problems. Monitoring bowel movements for constipation is not needed.

A nurse is conducting a class geared toward changes in early pregnancy and self-care items like perineal hygiene. A woman shares that she douches at least once a day since she has "so much discharge" from her vagina. Which response by the nurse is most appropriate at this time? "Douching will definitely keep your vagina clean." "During pregnancy, you should not douche because it can cause fluid to enter the cervix resulting in an infection." "If you prepare your own douching solution, be sure to boil the water to kill bacteria." "Let's discuss this with your health care provider before you continue douching."

"During pregnancy, you should not douche because it can cause fluid to enter the cervix resulting in an infection." Explanation: Even if vaginal discharge seems excessive, douching is contraindicated because the force of the irrigating fluid could cause the solution to enter the cervix, leading to a uterine infection. In addition, douching alters the pH of the vagina, leading to an increased risk of vaginal bacterial growth. Stating that douching will keep the client clean does not provide the client with the information she needs. Boiling water for a douche will not prevent development of infection. The nurse is capable of responding to the client directly without referring the client to the health care provider.

A woman comes to the prenatal clinic and undergoes a pelvic exam. The doctor notes a softening of the uterine isthmus. The nurse recognizes that this finding is known as what sign? Quickening Chadwick sign Goodell sign Hegar sign

Hegar sign Explanation: The Hegar sign is one of three signs that can be noted by a digital pelvic exam and involves the softening of the lower uterine segment. This is one of the probable signs of pregnancy, along with a positive Chadwick sign and Goodell sign.

A nurse is planning care for a newborn who is small for gestational age (SGA). Which of the following is the priority intervention the nurse should include in the newborn's plan of care? Monitor I&O. Monitor axillary temperature. Monitor blood glucose levels. Monitor weight.

Monitor blood glucose levels. Decreased stores of glycogen and a lower rate of gluconeogenesis place newborns who are SGA at higher risk for hypoglycemia. Monitoring of blood glucose levels is a priority intervention.

A nurse is caring for a middle adult client who has just received the diagnosis of endometrial cancer. In taking a nursing history, which of the following manifestations is likely to be reported by this client? Unilateral swelling on the posterior of the vulva Extreme abdominal pain with intercourse Green, malodorous vaginal discharge Postmenopausal bleeding

Postmenopausal bleeding Endometrial cancer involves cancerous growth of the endometrium (lining of the uterus). The most common manifestation of endometrial cancer is abnormal uterine bleeding, including postmenopausal bleeding and bleeding between normal periods in premenopausal women.

The partner of a pregnant client in her first trimester asks the nurse about the client's behavior recently, stating that she is very moody, seems happy one moment and is crying the next and all she wants to talk about is herself. What response would correctly address these concerns? What you are describing may be normal but we need to talk to her more in depth. Moodiness and irritability are not usual responses to pregnancy. Her body is changing and she may be angry about it. Pregnant women often experience mood swings and self-centeredness but this is normal.

Pregnant women often experience mood swings and self-centeredness but this is normal. Explanation: During the first trimester of pregnancy, the woman often has mood swings, bouts of irritability and is hypersensitive. The partner needs to know that these are all normal behaviors for a pregnant woman.

The nurse is admitting a client in labor. The nurse determines that the fetus is in a transverse lie by performing Leopold maneuvers. What intervention should the nurse provide for the client? Administer an analgesic to the client. Prepare the client for a cesarean birth. Prepare for a precipitous vaginal birth. Prepare to assist the care provider with an amniotomy.

Prepare the client for a cesarean birth. Explanation: If a transverse lie persists, the fetus cannot be born vaginally. Thus, the nurse will prepare the client for a caesarean birth. There is no indication the client will have precipitous labor. Amniotomy, artificial rupture of the membranes, is not indicated when preparing from a caesarean birth. The nurse would not administer analgesic before surgery unless prescribed by the health care provider.

The nurse is assessing a multipara client who presents to the hospital after approximately 2 hours of labor and notes the fetus is in a transverse lie. After notifying the RN and primary care provider, which action should the LPN prioritize? Prepare to assist with external version. Apply pressure to the client's lower back with a fisted hand. Assist with nitrazine and fern tests. Include a set of piper forceps when the table is prepped.

Prepare to assist with external version. Explanation: Transverse lie is a fetal malposition and is a cause for labor dystocia. The fetus would need to be turned to the occipital position using external version or be born via cesarean birth. Piper forceps are used in the birth of a fetus that is in the breech position. Nitrazine and fern tests are done to assess if amniotic fluid is leaking from the sac into the vagina. Counterpressure applied to the lower back with a fisted hand sometimes helps the client to cope with the "back labor" that is characteristic of occiput posterior (OP) positioning.

A nurse is assessing a newborn. Which of the following should the nurse understand is a clinical manifestation of pyloric stenosis? Absent bowel sounds Increased sodium levels Projectile vomiting after feedings Golf ball-sized mass over the left quadrant

Projectile vomiting after feedings. Pyloric stenosis is a narrowing and thickening of the pyloric canal between the stomach and the duodenum, resulting in projectile vomiting.

The nurse is reviewing an employer's guidelines to support pregnant employees. When reviewing the information, which guideline requires further follow-up and education by the nurse? Share information about health benefit coverage for support hose. Modify duties in later pregnancy for employees engaged in physical work. Promote overtime shifts to save money for parental leave. Provide a rest area for pregnant employees during breaks.

Promote overtime shifts to save money for parental leave. Explanation: Excessive overtime and working longer than 8-hour shifts are associated with an increase in preterm labor and other pregnancy complications; this is not recommended. Providing an area for rest supports the need for additional rest by pregnant employees. Support hose help to promote venous return and are useful for employees who spend a lot of time standing. Modifying duties during later pregnancy is important, as balance may be compromised and strenuous activity is not recommended.

A nurse is completing the admission assessment of a client who is at 38 weeks of gestation and has severe preeclampsia. Which of the following is an expected finding? Tachycardia Absence of clonus Polyuria Report of headache

Report of headache

What should you watch in a preterm newborn

Respirations, temperature, blood sugar(hypoglycemia), feeding reflex

A nurse is assessing a client who received magnesium sulfate to treat preterm labor. Which of the following clinical findings should the nurse identify as an indication of the toxicity of magnesium sulfate therapy and report to the provider? Respiratory depression Facial flushing Nausea Drowsiness

Respiratory depression

A nurse is preparing to administer magnesium sulfate IV to a client who is experiencing preterm labor. Which of the following is the priority nursing assessment for this client? Temperature Fetal heart rate (FHR) Bowel sounds Respiratory rate

Respiratory rate

A nurse in a provider's office is providing teaching to a client about modifiable risk factors for osteroporosis. Which of the following factors should the nurse include? (SATA) Sedentary lifestyle Obesity Carbonated beverages Caffeine intake Smoking tobacco products

Sedentary lifestyle Carbonated beverages Caffeine intake Smoking tobacco products Sedentary lifestyle is correct. The nurse should identify prolonged immobility or a sedentary lifestyle as a risk factor for the development of osteoporosis. Weight bearing and muscle activities are necessary for osteoblastic (bone building) activity. Obesity is incorrect. The nurse should identify obesity as a risk factor for the development of osteoarthritis. However, it is not a risk for the development of osteoporosis. Carbonated beverages is correct. The nurse should identify clients who consume more than 40 oz of carbonated beverages daily are at increased risk for calcium loss and the development of osteoporosis. Caffeine intake is correct The nurse should identify caffeine intake as a risk factor for the development of osteoporosis due to excessive losses of calcium in the urine. The body maintains homeostasis when this occurs by pulling calcium from the bones, making them more fragile and causing osteoporosis. Smoking tobacco products is correct. The nurse should identify tobacco use as a risk factor for the development of osteoporosis due to respiratory acidosis that can cause bone loss.

The nurse is assessing a client at her first prenatal visit and reports her LMP started December 1. Which date will the nurse predict for the EDD? July 7 October 7 September 8 August 8

September 8 Explanation: According to Naegele rule, the estimated date of birth is September 8. Add 7 days and subtract 3 months to the LMP to determine the estimated date of birth.

A pregnant client at 33 weeks' gestation is in the office for a routine visit. She lies down on her back and while the nurse is listening for fetal heart tones, the client tells the nurse that she feels lightheaded; her blood pressure is 82/58 mm Hg. What is the most likely explanation for this problem? She did not drink enough fluids prior to coming to the office. She is experiencing supine hypotension syndrome The baby is kicking her spinal column, causing a pinched nerve. Her hematocrit is low and she needs additional iron supplements.

She is experiencing supine hypotension syndrome Explanation: As the uterus gets larger toward the end of the pregnancy, it presses the aorta and vena cava against the spine, causing decreased blood return to the heart. This reduces cardiac output and the woman may feel lightheaded and dizzy and her blood pressure will drop.

A nurse in a clinic is assessing a client. Which of the following findings should the nurse identify as a risk factor for osteoporosis? Drinks one alcoholic beverage per day Smokes 1 pack of cigarettes per day Large body stature History of bone fracture during childhood

Smokes 1 pack of cigarettes per day. The nurse should identify active or passive smoking as a risk factor for osteoporosis.

A woman is concerned that orgasm will be harmful during pregnancy. Which statement is factual? Orgasm during pregnancy is potentially harmful. Some women experience orgasm intensely during pregnancy. Most women do not experience orgasm during pregnancy. Venous congestion in the pelvis makes orgasm painful.

Some women experience orgasm intensely during pregnancy. Explanation: Because of pelvic congestion, orgasm may be achieved more readily by pregnant women than nonpregnant women.

During late pregnancy, the nurse teaches a pregnant woman to lay on her left side to avoid what condition? Heartburn Supine hypotension syndrome Preeclampsia Frequent urination

Supine hypotension syndrome Explanation: The left side-lying position prevents the heavy uterus from resting on and compressing her vena cava, a condition known as supine hypotensive syndrome. Compression of the vena cava can cause maternal hypotension and poor gas exchange between the placenta and fetus. Preeclampsia is a condition characterized by elevated blood pressure and proteinuria. Once diagnosed, the treatment includes resting in a left-lateral position, but a side-lying position does not prevent preeclampsia. Urinary frequency in the third trimester is due to the enlarged uterus pressing on the bladder and is not influenced by position. Remaining in an upright position for 1 to 2 hours after meals helps to decrease heartburn.

The nurse-midwife is performing a pelvic examination on a client who came to her following a positive home pregnancy test. The nurse checks the woman's cervix for the probable sign of pregnancy known as Goodell sign. Which description illustrates this alteration? The lower uterine segment softens. The fundus enlarges. The cervix looks blue or purple when examined. The cervix softens.

The cervix softens. Explanation: At about the 8th week of gestation, the cervix softens, a probable sign known as Goodell sign. The cervix also looks blue or purple when examined; this is Chadwick sign, and may occur as early as the 6th week of pregnancy. At about 6 weeks, the lower uterine segment softens, a probable sign called Hegar sign. A softening of the uterine fundus, where the embryo has implanted, also occurs by about the 7th week, and the fundus enlarges by the 8th week.

A nurse is caring for an antepartum client whose laboratory findings indicate a negative rubella titer. Which of the following is the correct interpretation of this data? The client is not experiencing a rubella infection at this time. The client is immune to the rubella virus. The client requires a rubella vaccination at this time. The client requires a rubella immunization following delivery.

The client requires a rubella immunization following delivery. A negative rubella titer indicates that the client is susceptible to the rubella virus and needs vaccination following delivery. Immunization during pregnancy is contraindicated because of possible injury to the developing fetus. Following rubella immunization, the client should be cautioned not to conceive for 1 month.

When describing the role of a doula to a group of pregnant women, the nurse would include which information? The doula is capable of handling high-risk births and emergencies. The doula is a professionally trained nurse hired to provide physical and emotional support. The doula primarily focuses on providing continuous labor support. The doula can perform any necessary clinical procedures.

The doula primarily focuses on providing continuous labor support. Explanation: Doulas provide the woman with continuous support throughout labor. The doula is a laywoman trained to provide women and families with encouragement, emotional and physical support, and information through late pregnancy, labor, and birth. A doula does not perform any clinical procedures and is not trained to handle high-risk births and emergencies.

A nurse is observing a new mother bathing her newborn son for the first time. For which of the following actions should the nurse intervene? The mother cleans the newborn's eyes from the inner canthus outwards. The mother cleans the umbilical cord with tap water. The mother leaves the yellow exudate on the circumcision site. The mother plans to use a cotton-tipped swab to clean the nares.

The mother plans to use a cotton-tipped swab to clean the nares.

postterm pregnancy risks for mothers

Maternal: longer labor, injury to the perineum, C-section, dystocia, birth trauma, postpartum hemorrhage, and infection

The nurse is assisting with a G2P1, 24-year-old client who has experienced an uneventful pregnancy and is now progressing well through labor. Which action should be prioritized after noting the fetal head has retracted into the vagina after emerging? Use Zavanelli maneuver. Attempt to push in one of the fetus's shoulders. Use McRoberts maneuver. Apply pressure to the fundus.

Use McRoberts maneuver. Explanation: McRoberts maneuver intervention is used with a large baby who may have shoulder dystocia and requires assistance. The legs are sharply flexed by a support person or nurse, and the movement will help to open the pelvis to the widest diameter possible. Zavanelli maneuver is performed when the practitioner pushes the fetal head back in the birth canal and performs an emergency cesarean birth. Fundal pressure is contraindicated with shoulder dystocia. It is outside the scope of practice for the LPN to attempt birth of the fetus by pushing one of the fetus' shoulders in a clockwise or counterclockwise motion.

A nurse is caring for a client who is 5 hr postpartum following a vaginal birth of a newborn weighing 9 lb 6 oz (4252g). The nurse should recognize that this client is at risk for which of the following postpartum complications? A. Puerperal infections B. Retained placental fragments C. Thrombophlebitis D. Uterine atony

Uterine atony A uterus that is over distended, such as from a macrosomic fetus, has an increased risk of uterine atony.

What are some characteristics of a preterm newborn

Weight under 5.5 pounds Poor muscle tone Minimal subq Undescended testes Lanugo Poorly formed ear pinna Fused eyelids Soft spongy skull bones Matted scalp hair Absent or few creases in soles and palms Minimal scrotal Rugae Prominent labia and clitoris Thin transparent skin Abundant vernix

uterine rupture risk factors

Who is at risk- prior C-sections, large babies, contractions too close to each other, multiple babies, rapid labor, prior abortion/uterine manipulation with window or scarring in uterus.

A nurse is caring for a newborn who is small for gestational age (SGA). Which of the following findings is associated with this condition? Moist skin Protruded abdomen Gray umbilical cord Wide skull sutures

Wide skull sutures Newborns who are SGA have wide skull sutures due to inadequate bone growth. Head circumference is smaller than in a normal newborn and there is reduced brain capacity.

A nurse is teaching a client who is at 23 weeks of gestation about immunizations. Which of the following statements should the nurse include in the teaching? "You should not receive the rubella vaccine while breastfeeding." "You should receive a varicella vaccine before you deliver." "You can receive an influenza vaccination during pregnancy." "You cannot receive the Tdap vaccine until after you deliver."

You can receive an influenza vaccination during pregnancy.

A nurse assesses a client in labor and suspects dysfunctional labor (hypotonic uterine dysfunction). The woman's membranes have ruptured and fetopelvic disproportion is ruled out. Which intervention would the nurse expect to include in the plan of care for this client? providing a comfortable environment with dim lighting encouraging the woman to assume a hands-and-knees position preparing the woman for an amniotomy administering oxytocin

administering oxytocin Explanation: Oxytocin would be appropriate for the woman experiencing dysfunctional labor (hypotonic uterine dysfunction). Comfort measures minimize the woman's stress and promote relaxation so that she can work more effectively with the forces of labor. An amniotomy may be used if the membranes were intact. It may also be used with hypotonic uterine dysfunction to augment labor. A hands-and-knees position helps to promote fetal head rotation with a persistent occiput posterior position.

After teaching a review class to a group of perinatal nurses about various methods for cervical ripening, the nurse determines that the teaching was successful when the group identifies which method as surgical? breast stimulation amniotomy laminaria prostaglandin

amniotomy Explanation: Amniotomy is considered a surgical method of cervical ripening. Breast stimulation is considered a nonpharmacologic method for ripening the cervix. Laminaria is a hygroscopic dilator that mechanically causes cervical ripening. Prostaglandins are pharmacologic methods for cervical ripening.

A woman whose fetus is in the occiput posterior position is experiencing increased back pain. Which is the best way for the nurse to help alleviate this back pain? applying a heating pad to the back applying ice to the back applying counterpressure to the back performing acupuncture on the back

applying counterpressure to the back Explanation: Counterpressure applied to the lower back with a fisted hand sometimes helps the woman cope with "back labor" associated with occiput-posterior positioning. The others are not recommended or used techniques for a woman in labor with back pain.

A nurse at the health care facility assesses a client at 20 weeks' gestation. The client is healthy and progressing well, without any sign of complications. Where should the nurse expect to measure the fundal height in this client? halfway between the symphysis pubis and the umbilicus at the top of the symphysis pubis at the xiphoid process at the level of the umbilicus

at the level of the umbilicus Explanation: In the 20th week of gestation, the nurse should expect to find the fundus at the level of the umbilicus. The nurse should palpate at the top of the symphysis pubis between 10 to 12 weeks' gestation. At 16 weeks' gestation, the fundus should reach halfway between the symphysis pubis and the umbilicus. With a full-term pregnancy, the fundus should reach the xiphoid process.

Which assessment finding in the pregnant woman at 12 weeks' gestation should the nurse find most concerning? The inability to: feel fetal movements. detect fetal heart sounds with a Doppler. palpate the fetal outline. hear the fetal heartbeat with a stethoscope.

detect fetal heart sounds with a Doppler. Explanation: Fetal heart sounds are audible with a Doppler at 10 to 12 weeks' gestation but cannot be heard through a stethoscope until 18 to 20 weeks' gestation. Fetal movements can be felt by a woman as early as 16 weeks of pregnancy and felt by the examiner around 20 weeks' gestation. The fetal outline is also palpable around 20 weeks' gestation.

The nurse is describing pregnancy danger signs to a pregnant woman who is in her first trimester. Which danger sign might occur at this point in her pregnancy? swelling of extremities excessive vomiting dyspnea lower abdominal pressure

excessive vomiting Explanation: Excessive vomiting is a warning sign in the first trimester. Dyspnea, lower abdominal pressures, and swelling of face or extremities may occur late in pregnancy.

A fetus is experiencing shoulder dystocia during birth. The nurse would place priority on performing which fetal assessment postbirth? assess for cleft palate brachial plexus assessment monitor for a cardiac anomaly extensive lacerations

brachial plexus assessment Explanation: The nurse should identify nerve damage as a risk to the fetus in cases of shoulder dystocia. Other fetal risks include asphyxia, clavicle fracture, central nervous system injury or dysfunction, and death. Extensive lacerations is a poor maternal outcome due to the occurrence of shoulder dystocia, which should be assessed and treated. Cleft palate and cardiac anomalies are not related to shoulder dystocia.

HSV management of laboring mother

c section Mom will be on acyclovir - PO daily If baby exposed, baby will be given same drug to take home and take PO daily

What is the major concern for a lactose intolerant woman who is pregnant? dangerous symptom of abdominal cramping calcium deficiency nausea and vomiting vitamin D deficiency

calcium deficiency Explanation: Calcium deficiency is a major concern for the pregnant woman who is lactose intolerant. There are several ways to address this concern. Some lactose-intolerant individuals are able to tolerate cooked forms of milk, such as pudding or custard. Cultured or fermented dairy products, such as buttermilk, yogurt, and some cheeses may also be tolerated. A chewable lactase tablet may be taken with milk. Lactase-treated milk is available in most supermarkets and may be helpful. Other options are to drink calcium-enriched orange juice or soy milk or to take a calcium supplement. If the woman is infrequently exposed to sunlight, she will need a vitamin D supplement.

The nurse is teaching about an iron supplement that the client is going to take every day. The nurse teaches the client to take the iron supplement with which type of fluid? low-fat milk hot tea citrus juice ice water

citrus juice Explanation: The citric acid in juice enhances absorption of iron in the GI tract. Ice water and tea do not enhance iron absorption, and milk can inhibit iron absorption.

Which intervention would be most important when caring for the client with breech presentation confirmed by ultrasound? auscultating the fetal heart rate at the level of the umbilicus applying suprapubic pressure against the fetal back noting the space at the maternal umbilicus continuing to monitor maternal and fetal status

continuing to monitor maternal and fetal status Explanation: Once a breech presentation is confirmed by ultrasound, the nurse should continue to monitor the maternal and fetal status when the team makes decisions about the method of birth. The nurse usually plays an important role in communicating information during this time. Applying suprapubic pressure against the fetal back is the nursing intervention for shoulder dystocia and may not be required for breech presentation. Noting the space or dip at the maternal umbilicus and auscultating the fetal heart rate at the umbilicus level are assessments related to occipitoposterior positioning of the fetus.

During pregnancy a woman has many psychological adaptations that must be made. The nurse must remember that the baby's father is also experiencing the pregnancy and has adaptations that must be made. Some fathers actually have symptoms of the pregnancy along with the mothers. What is this called? pseudo pregnancy pregnancy syndrome couvade syndrome cretinism

couvade syndrome Explanation: Some fathers actually experience some of the physical symptoms of pregnancy, such as nausea and vomiting, along with their partner. This phenomenon is called couvade syndrome.

Which disease process would the nurse screen for under potential genetic disorders? tuberculosis cystic fibrosis rheumatic fever asthma

cystic fibrosis Explanation: Screening of genetically linked disorders is important when obtaining a family history. Cystic fibrosis is a genetically linked disorder. Tuberculosis is an infectious disorder. Rheumatic fever stems from a streptococcus infection. Asthma is a hypersensitivity typically from an environmental allergy.

A nurse is taking a history during a client's first prenatal visit. Which assessment finding would alert the nurse to the need for further assessment? maternal age of 28 years history of occasional use of OTC pain relievers history of diabetes for 4 years history of exercising twice a week

history of diabetes for 4 years Explanation: A diagnosis of diabetes in a pregnant client increases risk for both the client and the infant during pregnancy and requires close monitoring and follow-up. This client's age, exercise history, and history of occasional OTC pain reliever use do not increase pregnancy risk.

precipitus labor

labor that lasts less than 2 hours

The nurse is admitting a client at 23 weeks' gestation in preparation for induction and delivery after it was determined the fetus had died secondary to trauma. When asked by the client to explain what went wrong, the nurse can point out which potential cause for this loss? placental abruption genetic abnormality preeclampsia premature rupture of membranes

placental abruption Explanation: The most common cause of fetal death after a trauma is placental abruption (abruptio placentae), where the placenta separates from the uterus, and the fetus is not able to survive. Genetic abnormalities typically cause spontaneous abortion (miscarriage) in the first trimester. Trauma does not cause preeclampsia (which is related to various issues in the mother) nor does trauma usually cause PROM.

Which client immunization titer is most important to assess and document in the prenatal record of the pregnant woman? rotavirus polio rubella diphtheria

rubella Explanation: Rubella (German measles) is an infection caused by the rubella virus. The virus causes a rash and mild symptoms in children but can be teratogenic to a fetus. A rubella titer determines if the mother is immune to the virus. If the mother is not immune, she will receive a rubella immunization immediately after delivery. Diphtheria and polio are infant vaccines but not as teratogenic to the fetus. Rotavirus is a gastrointestinal virus typically mild in adults.

what to do in the case of meconium aspiration

tracheal suctioning Umbilical arterial line and venous catheter High levels of oxygen; high-pressure ventilation Exogenous surfactant Prophylactic antibiotics Assess for signs of distress and complications

A client with a pendulous abdomen and uterine fibroids (uterine myomas) has just begun labor and arrived at the hospital. After examining the client, the primary care provider informs the nurse that the fetus appears to be malpositioned in the uterus. Which fetal position or presentation should the nurse most expect in this woman? occipitoposterior position anterior fetal position cephalic presentation transverse lie

transverse lie Explanation: A transverse lie, in which the fetus is more horizontal than vertical, occurs in the following instances: women with pendulous abdomens; uterine fibroids (uterine myomas) that obstruct the lower uterine segment; contraction of the pelvic brim; congenital abnormalities of the uterus; or hydramnios. Anterior fetal position and cephalic presentation are normal conditions. Occipitoposterior position tends to occur in women with android, anthropoid, or contracted pelvis.

The nurse is reinforcing health care provider education on the technique for an amniocentesis. Which piece of equipment will the nurse have ready? sterile field with scalpel ultrasound equipment sterile urine cup Foley catheter

ultrasound equipment Explanation: First, the health care provider identifies a pocket of amniotic fluid using an ultrasound machine. A scalpel is not used in the procedure. A urine culture is not obtained prior to the procedure nor is a Foley catheter inserted.

The nurse is assessing a primigravida woman at a routine prenatal visit. Which assessment finding is reinforcing to the client that she is definitely pregnant? uterine growth positive hCG blood result ultrasound picture of her fetus continued amenorrhea

ultrasound picture of her fetus Explanation: A positive sign of pregnancy is visualization of the fetus by ultrasound at 6+ weeks. Amenorrhea is a presumptive sign and can be caused by a variety of factors. Positive hCG in the blood and uterine growth are both probable signs but can be caused by hydatidiform or tumors.

A woman in active labor has just had her membranes ruptured to speed up labor. The nurse is concerned the woman is experiencing a prolapse of the umbilical cord when the nurse notices which pattern on the fetal heart monitor? variable deceleration pattern early deceleration with each contraction late deceleration with late recovery following contraction fetal heart rate (FHR) increase to 200 beats/min

variable deceleration pattern Explanation: Umbilical cord prolapse can be seen after the membranes have ruptured, when the FHR is displaying a sudden variable deceleration FHR pattern on a fetal monitor. It is not uncommon for FHR to increase following a procedure. Early deceleration with each contraction is seen when the fetal head is being compressed through the pelvic opening. Late deceleration with late recovery following contraction is associated with uteroplacental insufficiency (UPI).

The nurse assesses that a fetus is in an occiput posterior position. The nurse predicts the client will experience which situation related to this assessment? Need to have the baby manually rotated Shorter dilation (dilatation) stage of labor Experience of additional back pain Necessity for vacuum extraction for birth

Experience of additional back pain Explanation: Most women whose fetus is in a posterior position experience back pain while in labor. Pressure against the back by a support person often reduces this type of pain. An occiput posterior position does not make for a shorter (dilation) dilatation stage of labor. OP position does not indicate the need to have the baby manually rotated, nor does it indicate a necessity for a vacuum extraction birth.

What can you give to help with preterm labor contractions to help stop contractions?

Nifedipine (PO calcium channel blocker) and Magnesium Sulfate (IV)

nursing care for postterm pregnancy

Nursing care includes good labor management with careful fetal assessment, assist with amnioinfusion for meconium, early skin-to-skin and feeding of newborn, monitor newborn for hypoglycemia

A nurse is caring for a client who is at 40 weeks of gestation and is in labor. The clients ultrasound examination indicates that the fetus is small for gestational age. Which of the following interventions should be included in the newborns plan of care? Observe for meconium in respiratory secretions. Monitor for hyperglycemia. Identify manifestations of anemia. Monitor for hyperthermia.

Observe for meconium in respiratory secretions. When a fetus is SGA, there is an increased risk for intrauterine hypoxia due to the presence of meconium in the amniotic fluid. The nurse should observe for meconium in respiratory secretions when suctioning the newborn at delivery. Newborns who are SGA are at risk for perinatal asphyxia due to the stress of labor and are often depressed. They require careful resuscitation and suctioning at delivery.

A woman presents at Labor and Delivery very upset. She reports that she has not felt her baby moving for the last 6 hours. The nurse listens for a fetal heart rate and cannot find a heartbeat. An ultrasound confirms fetal death and labor induction is started. What intervention by the nurse would be appropriate for this mother at this time? Offer to take pictures and footprints of the infant once it is delivered. Recommend that she not hold the infant after it is delivered so as to not upset her more. Explain to her that there was probably something wrong with the infant and that is why it died. Call the hospital chaplain to talk to the parents.

Offer to take pictures and footprints of the infant once it is delivered. Explanation: When parents are faced with a fetal death, they need comfort and support without being intrusive. Taking pictures, footprints and gathering other mementos are very important in helping the family deal with the death. The mother is encouraged to hold the infant after delivery and name it. Telling the parents that the infant was probably defective is hurtful and not supportive to them. Calling the hospital chaplain is something that can be offered but should not be done without the parent's approval.

Preterm Premature Rupture of Membrane (PPROM)

a rupture of the membranes before labor begins if PROM occurs before 37 wks pregnancy

Uterine Rupture

a tear in the wall of the uterus Obstetric emergency; onset marked by sudden fetal bradycardia

Placental abruption

premature separation of the placenta from the uterus Complete or partial after 20 weeks

A nurse is teaching a client who is 30 weeks' pregnant about ways to deal with pyrosis (heartburn). The nurse determines a need for additional teaching based on which client statement? "I should lie down for 1/2 hour after eating." "I need to raise the head of my bed about 15 to 30 degrees." "I should chew my food slowly." "I need to cut out caffeine."

"I should lie down for 1/2 hour after eating." Explanation: The client should remain sitting for 1 to 3 hours after eating and avoid lying down within 3 hours of eating. Cutting out caffeine, chewing food slowly, and raising the head of the bed are helpful in reducing pyrosis (heartburn) of pregnancy.

A nurse is teaching about crib safety with the parent of a newborn. Which of the following statements by the client indicates understanding of the teaching? "I will place my baby on his stomach when he is sleeping." "I should remove extra blankets from my baby's crib." "I should pad the mattress in my baby's crib so that he will be more comfortable when he sleeps." "I should place my baby's crib next to the heater to keep him warm during the winter."

"I should remove extra blankets from my baby's crib." Loose bedding such as sheets and blankets could cover the baby's head and lead to suffocation.

A nurse is teaching about nutrition guidelines to a parent of a newborn. Which of the following statement by the parent indicates understanding of the teaching? "I should start solid foods when my baby is 3 months old." "I should introduce cow's milk when my baby is 9 months old." "I should wait to give fruit juice until my baby is 6 months of age." "I should wait to begin fluoride supplements until my baby is 4 months of age."

"I should wait to give fruit juice until my baby is 6 months of age." Fruit juice provides minimal nutritional value to the infant's diet. Therefore, fruit juices should be limited and not offered until the infant is 6 months of age.

A nurse is providing teaching about phenylketonuria (PKU) testing to the parent of a newborn. Which of the following statements by the parent indicates a need for additional teaching? "My baby will be placed under special lights if the test result is positive." "My baby needs to be on formula or breast milk before the test can be done." "This test checks for a genetic disorder that can be managed by diet." "Sometimes the test is repeated in the doctor's office at the baby's 2-week check-up."

"My baby will be placed under special lights if the test result is positive."

The nurse is teaching the pregnant woman about nutrition for herself and her baby. Which statement by the woman indicates that the teaching was effective? "I can eat any seafood that I like because it contains phosphorus, which is a nutrient that pregnant women need." "I will need to take iron supplementation throughout my pregnancy even if I am not anemic." "Milk production requires higher levels of calcium; therefore, if I am going to breastfeed, I must take a calcium supplement during pregnancy." "Because I am pregnant, I can eat anything I want and not worry about weight gain."

"I will need to take iron supplementation throughout my pregnancy even if I am not anemic." Explanation: Iron is recommended for all pregnant women because it is almost impossible for the pregnant woman to get what is required from diet alone, especially after 20 weeks' gestation when the requirements of the fetus increase. Pregnant women can get many nutrients from seafood including phosphorus, but there are specific recommendations about types of fish to avoid because of the risk of mercury poisoning. Milk production actually requires higher levels of zinc, which can be obtained from a healthy diet. Calcium requirements do not increase above prepregnancy levels during pregnancy because calcium absorption is enhanced during pregnancy. It can be unsafe for the pregnant woman to eat anything she wants and gain too much weight. A woman who gains too much weight during pregnancy is at risk for delivering a macrosomic baby.

Which statement by a pregnant client would validate that she understood the nurse's teaching regarding the risk of venous stasis during pregnancy? "If I lie down on my back, I will be less likely to develop blood clots in my legs." "My clotting times and bleeding times will be unchanged during this pregnancy even though I am at higher risk of clots in my leg vein." "The increase in my blood volume and increased clotting factors can cause me to bleed more at delivery." "My clotting factors will be unchanged during pregnancy but I must be careful to not cross my legs at anytime."

"My clotting times and bleeding times will be unchanged during this pregnancy even though I am at higher risk of clots in my leg vein." Explanation: Increased levels of fibrinogen make pregnancy a hypercoagulable state. Clotting times and bleeding times are unchanged, however. The pregnant woman does run the risk of development of a venous thrombosis as the uterus enlarges and inhibits venous return from the lower extremities due to the hypercoagulable state and venous stasis.

A nurse is teaching a client who has herpes simplex virus type 2 about disease transmission. Which of the following statements by the client indicates an understanding of the teaching? "Itching or tingling occurs at the site where a blister forms." "It is okay to share towels as long as it belongs to a family member." "It is not contagious after the blisters rupture." "I will take medication daily to prevent more sores."

"Itching or tingling occurs at the site where a blister forms." The nurse should instruct the client that itching or tingling will occur 1 to 2 days before the blister appears in herpes simplex virus type 2.

The nurse is caring for a client at 8 weeks' gestation who states, "I did not plan for this right now and I am not happy or excited about this pregnancy. I am not sure what to do." Which response by the nurse is best? "Many women feel this way during the first trimester." "Do not worry. Once you hold this baby, everything will be fine." "You will become excited and happy when you feel the baby move." "We can refer you to a clinic for potential termination if you desire."

"Many women feel this way during the first trimester." Explanation: The best response is to let the client know this is a common feeling among all pregnant women. Most women experience ambivalence during the first trimester whether the pregnancy was planned or not. Acceptance of the pregnancy commonly occurs during the second trimester when quickening, or feeling the baby move, occurs. However, it is not appropriate for the nurse to assume the client will become excited as each pregnancy is unique and a time of dramatic alterations. Stating not to worry and everything will be fine is nontherapeutic communication and does not focus on the client's concern. The nurse would discuss the client's feelings and concerns before making a referral.

A nurse is teaching a group of clients who are in their first trimester about exercise during pregnancy. Which of the following statements should the nurse include in the teaching? "Refrain from exercises that include stretching." "Moderate exercise improves circulation." "It is recommended to increase your weight-bearing exercises." "It is recommended to rest for 30 minutes before each new exercise."

"Moderate exercise improves circulation." Improving circulation is just one of the many benefits of moderate exercise during pregnancy. It enhances well-being, promotes rest and relaxation, and improves muscle tone.

A woman has been in labor for the past 8 hours, and she has progressed to the second stage of labor. However, after 2 hours with no further descent, the provider diagnoses "arrest of labor." The woman asks, "Why is this happening?" Which response is the best answer to this question? "It is likely that your body has not secreted enough hormones to soften the ligaments so your pelvic bones can shift to allow birth of the baby." "More than likely you have cephalopelvic disproportion (CPD) where baby's head cannot make it through the canal." "Maybe your uterus is just tired and needs a rest." "Maybe your baby has developed hydrocephaly and the head is too swollen."

"More than likely you have cephalopelvic disproportion (CPD) where baby's head cannot make it through the canal." Explanation: Arrest of labor results when no descent has occurred for 2 hours in a nullipara or 1 hour in a multipara. The most likely cause for arrest of descent during the second stage is CPD. Rest should allow the uterine contractions to be more efficient. The hormones secreted during pregnancy allow ligaments to soften so bones can shift to allow birth. Ultrasound would have previously been diagnosed prior to the onset of labor.

The nurse is teaching a prenatal class about preparing for their expanding families. What is helpful advice from the nurse? "Expect your other children to react positively to their new brother/sister." "The hormones of pregnancy may cause anxiety or depression postpartum." "Your old coping methods will adequately get you through this period of adjustment." "Caring for your new infant is instinctual and will come naturally to you."

"The hormones of pregnancy may cause anxiety or depression postpartum." Explanation: The "raging hormones" of pregnancy can keep the woman slightly out of touch with her usual methods of coping. Although she may normally interact and communicate in quite mature ways, during a pregnancy she may become depressed, anxious, withdrawn, or angry as she accomplishes her own developmental tasks. Siblings often react to a pregnancy by regression in behavior and attitude because they fear they will be replaced or unloved. In addition to anticipatory guidance concerning the alterations in family structure and functioning, prenatal preparation for first-time parents involves learning the basics of infant care and preparing for infant feeding, particularly for women who plan to breastfeed.

A nurse is caring for a client who has rubella at the time of delivery and asks why her newborn is being placed in isolation. Which of the following responses by the nurse is appropriate? "The newborn might be actively shedding the virus." "The newborn is at risk for developing a TORCH infection." "The child might develop encephalitis, a complication of rubella." "Exposure to rubella will suppress the newborn's immune response."

"The newborn might be actively shedding the virus." Infants born to mothers who have rubella will continue to shed the rubella virus for up to 18 months postdelivery.

.A nurse is caring for a client who delivered a healthy term newborn via cesarean birth. The client asks the nurse, "Is there a chance that I could deliver my next baby without having a cesarean section?" Which of the following responses should the nurse provide? "The primary consideration is what type of incision was performed this time." "There are so many variables that you'll have to ask your obstetrician." "It's too soon for you to be worrying about this now." "A repeat cesarean birth is safer for both you and your baby."

"The primary consideration is what type of incision was performed this time." Rationale: The most common type of incision during a cesarean birth is transverse, which is made across the lower, thinner part of the uterus. It is the primary criteria that permits a vaginal birth after a cesarean (VBAC). Other types of incisions increase the risk of uterine rupture. Additional criteria for VBAC include an adequate maternal pelvis, no uterine scars or history of rupture, the availability of a provider to monitor labor, and personnel to perform a cesarean birth if needed.

A pregnant client reports an increase in a thick, whitish vaginal discharge. Which response by the nurse would be most appropriate? "You need to be assessed for a fungal infection." "You should refrain from any sexual activity." "Use a local antifungal agent regularly." "This discharge is normal during pregnancy."

"This discharge is normal during pregnancy." Explanation: During pregnancy, vaginal secretions become more acidic, white, and thick. Most women experience an increase in a whitish vaginal discharge, called leukorrhea, during pregnancy. The nurse should inform the client that the vaginal discharge is normal except when it is accompanied by itching and irritation, possibly suggesting Candida albicans infection (a monilial vaginitis), which is a very common occurrence in this glycogen-rich environment. Monilial vaginitis is a benign fungal condition and is treated with local antifungal agents. The client need not refrain from sexual activity when there is an increase in a thick, whitish vaginal discharge.

When providing preconception care to a client, which instruction will the nurse to provide about medications during pregnancy? "You need to avoid all prescription, over-the-counter, and herbal medications when you are pregnant." "You should switch to herbal remedies because they are safer to use than other types medicines." "It is safe for you to take over-the-counter medications." "You need to talk with your health care provider about using all prescription, over-the-counter, and herbal medications."

"You need to talk with your health care provider about using all prescription, over-the-counter, and herbal medications." Explanation: Medication use is common during pregnancy, with prevalence estimates generally exceeding 65% and increasing over the years. Pregnant women use a wide variety of both prescription and over-the-counter medications for both pregnancy-related conditions and conditions unrelated to pregnancy conditions. Little is known about the effects of taking most medications during pregnancy. It is best for pregnant women to not take any medications during their pregnancy. At the very least, they should be encouraged to discuss with the health care provider their current medications and any herbal remedies they take so that they can learn about any potential risks should they continue to take them during pregnancy. A common concern of many pregnant women involves the use of over-the-counter medications and herbal agents. Many women consider these products benign simply because they are available without a prescription. Although herbal medications are commonly thought of as "natural" alternatives to other medicines, they can be just as potent as some prescription medications. The nurse should encourage pregnant women to check with their health care providers before taking anything.

A nurse in the emergency department is admitting a client who is at 40 weeks of gestation, has ruptured membranes, and the nurse observes the newborns head is crowning. The client tells the nurse she wants to push. Which of the following statements should the nurse make? "You should go ahead and push to assist the delivery." "You should try to pant as the delivery proceeds." "You should try to perform slow-paced breathing." "You should take a deep, cleansing breath and breathe naturally."

"You should try to pant as the delivery proceeds." Panting allows uterine forces to expel the fetus and permits controlled muscle expansion to avoid rapid expulsion of the fetal head.

what gestation determines preterm labor?

- preterm labor is less than or equal to 36 weeks. - Late preterm (34 wks, 0 days - 36 wks, 6 days) - Early term (37 wks, 0 days - 38 wks, 6 days)

A client's membranes have just ruptured. Her fetus is presenting breech. Which action should the nurse do immediately to rule out prolapse of the umbilical cord in this client? Administer oxygen at 10 L/min by face mask. Place the woman in Trendelenburg position. Administer amnioinfusion. Assess fetal heart sounds.

Assess fetal heart sounds. Explanation: To rule out cord prolapse, always assess fetal heart sounds immediately after rupture of the membranes whether this occurs spontaneously or by amniotomy, as the fetal heart rate will be unusually slow or a variable deceleration pattern will become apparent when cord prolapse has occurred. The other answers refer to therapeutic interventions to implement once cord prolapse has been confirmed.

A nurse admits a woman who is at 38 weeks of gestation and in early labor with ruptured membranes. The nurse determines that the client's oral temperature is 38.9C (102F). Besides notifying the provider, which of the following is an appropriate nursing action? Recheck the client's temperature in 4 hr. Administer glucocorticoids intramuscularly. Assess the odor of the amniotic fluid. Prepare the client for emergency cesarean section.

Assess the odor of the amniotic fluid.

A nurse is caring for a preterm newborn who is in an incubator to maintain a neutral thermal environment. The father of the newborn asks the nurse why this is necessary. Which of the following responses should the nurse make? A. "Preterm newborns have a smaller body surface area than normal newborns." B. "The added brown fat layer in a preterm newborn reduces his ability to generate heat." C. "Preterm newborns lack adequate temperature control mechanisms." D. "The heat in the incubator rapidly dries the sweat of preterm newborns."

C. "Preterm newborns lack adequate temperature control mechanisms." Preterm newborns have poor body control of temperature and need support to avoid losing heat. They require an external heat source, such as an incubator.

A nurse is caring for a client who is postpartum. The client tells the nurse that the newborn's maternal grandmother was born deaf and asks how to tell if her newborn hears well. Which of the following statements should the nurse make? A. "There is no need to worry about that. Most forms of hearing loss are not inherited." B. "Look at how she looks at you when you speak. That's a good sign." C. "We do routine hearing screenings on newborns. You'll know the results before you leave the hospital." D. "The best way to determine if your baby can hear is to clap your hands loudly and see if she startles."

C. "We do routine hearing screenings on newborns. You'll know the results before you leave the hospital." Most states mandate hearing screening for all newborns. The two tests in use do not diagnose hearing loss, but determine whether or not a newborn requires further evaluation.

A 25-year-old client at 27 weeks' gestation reports waking up with leg cramps. Which suggestion should the nurse point out to the client to help relieve this discomfort? Encourage her to drink more fluids, 10 glasses a day. Dorsiflex the foot while extending her leg during the cramp. Avoid any supplementation of vitamins or minerals. Use plantar flexion exercises three times every day.

Dorsiflex the foot while extending her leg during the cramp. Explanation: Plantar flexion can make cramps worse, so dorsiflexion while extending the leg can relieve the cramp; excess fluid and lack of supplementation with vitamins or minerals may worsen cramps. Performing plantar flexion exercise does not prevent the cramp. Increasing fluids may help, but has never proven to eliminate cramping.

A nurse is completing discharge teaching to client in her 35th week of pregnancy who has mild preeclampsia. Which of the following information about nutrition should be included in the teaching? Consume 40 to 50 g of protein daily. Avoid salting of foods during cooking. Drink 48 to 64 ounces of water daily. Limit intake of whole grains, raw fruits, and vegetables.

Drink 48 to 64 ounces of water daily.

A client in her 39th week of gestation arrives at the maternity clinic stating that earlier in her pregnancy, she experienced shortness of breath. However, for the past few days, she has been able to breathe easily, but she has also begun to experience increased urinary frequency. A nurse is assigned to perform the physical examination of the client. Which observation is most likely? The fundus is at the level of the umbilicus and measures 20 cm. The lower uterine segment and cervix have softened. Fundal height has dropped since the last recording. Fundal height is at its highest level at the xiphoid process.

Fundal height has dropped since the last recording. Explanation: Between 38 and 40 weeks of gestation, the fundal height drops as the fetus begins to descend and engage into the pelvis. Because it pushes against the diaphragm, many women experience shortness of breath. By 40 weeks, the fetal head begins to descend and engage into the pelvis. Although breathing becomes easier because of this descent, the pressure on the urinary bladder now increases, and women experience urinary frequency. The fundus reaches its highest level at the xiphoid process at approximately 36, not 39, weeks. By 20 weeks' gestation, the fundus is at the level of the umbilicus and measures 20 cm. At between 6 and 8 weeks of gestation, the cervix begins to soften (Goodell sign) and the lower uterine segment softens (Hegar's sign).

A 41-year-old pregnant woman and her husband are anxiously awaiting the results of various blood tests to evaluate the fetus for potential Down syndrome, neural tube defects, and spina bifida. Client education should include which information? Treatment can be started once the test results are back. The blood tests are definitive. A second set of screening tests can be obtained to confirm results. Further testing will be required to confirm any diagnosis.

Further testing will be required to confirm any diagnosis. Explanation: Nursing management related to marker screening tests consists primarily of providing education about the tests. Remind the couple that a definitive diagnosis is not made without further tests such as an amniocentesis. The blood tests are not definitive but only strongly suggest the possibility of a defect. For some conditions there are no treatments. The couple may request a second set, but the health care provider will probably suggest proceeding with the more definitive methods to confirm the diagnosis.

A nurse is collecting data during an admission assessment of a client who is pregnant with twins. The client has a 4-year-old child who was delivered at 38 weeks' gestation and tells the nurse that she does have a history of spontaneous abortion (miscarriage) within the first trimester. The nurse is correct to document the history as: G = 1, T = 1, P = 1, A = 0, L = 1 G = 3, T = 1, P = 0, A = 1, L = 1 G = 2, T = 0, P = 0, A = 0, L = 1 G = 4, T = 2, P = 0, A = 0, L = 1

G = 3, T = 1, P = 0, A = 1, L = 1 Explanation: The GTPAL stands for Gravida -- number of pregnancies, which is 3 (current, 4-year-old, and miscarriage); Term -- only one pregnancy thus far carried to term; Preterm deliveries -- 0; Abortions (either elective or miscarriage) -- 1; Living children -- 1. Do not be distracted by the twins. That is still one pregnancy.

A multigravida client is pregnant for the third time. Her previous two pregnancies ended in an abortion in the first and third month of pregnancy. How will the nurse classify her pregnancy history? G3 P0020 G2 P0020 G2 P1020 G3 P0021

G3 P0020 Explanation: Gravida (G) is the total number of pregnancies she has had, including the present one. Therefore she is G3 and not G2. Para (P), the outcome of her pregnancies, is further classified by the FPAL system as follows: F = Full term: number of babies born at 37 or more weeks of gestation, which is 0 and not 1 in this case. P = Preterm: number of babies born between 20 and 37 weeks of gestation, which is 0 in this case. A = Abortions: total number of spontaneous and elective abortions, which is 2 in this case. L = Living children, as of today. She has no living children; therefore, it is 0 and not 1.

number one risk factor for placental abruption

HTN

Placental abruption risk factors

HTN (causes vasoconstriction that alerts the placenta to separate), trauma (seatbelts in MVC), drug use, previous abruption, smoking, PROM, multifetal pregnancy

What actions should be taken for in newborn with a mother who has diabetes

Monitor for hypoglycemia***, hypocalcemia, hypomagnesemia, hyperbilirubinemia, RDS Watch for birth trauma Mild hypoglycemia should have early feedings Administer IV glucose if needed

what is important with ANY vaginal bleeding

NOTHING IN THE VAGINA. no cervical exams. nothing.

A nurse is caring for a newborn who has respiratory depression. Which of the following medications should the nurse anticipate administering? Flumazenil Physostigmine Terbutaline Naloxone

Naloxone Naloxone is an opioid antagonist and is administered to reverse opioid toxicity or reverse neonatal respiratory depression. Dosage for a newborn is 0.01 mg/kg, and is repeated every 2 to 3 min until adequate respiratory function returns.

postterm pregnancy risks for neonates

Neonate: meconium aspiration, hypoglycemia, poor fetal growth, poor placental perfusion, shoulder dystocia, macrosomia & stillbirth

A client in her 29th week of gestation reports dizziness and clamminess when assuming a supine position. During the assessment, the nurse observes there is a marked decrease in the client's blood pressure. Which intervention should the nurse implement to help alleviate this client's condition? Place the client in the left lateral position. Keep the client's legs slightly elevated. Place the client in an orthopneic position. Keep the head of the client's bed slightly elevated.

Place the client in the left lateral position. Explanation: The symptoms experienced by the client indicate supine hypotension syndrome. When the pregnant woman assumes a supine position, the expanding uterus exerts pressure on the inferior vena. The nurse should place the client in the left lateral position to correct this syndrome and optimize cardiac output and uterine perfusion. Elevating the client's legs, placing the client in an orthopneic position, or keeping the head of the bed elevated will not help alleviate the client's condition.

A client in her 29th week of gestation reports dizziness and clamminess when assuming a supine position. During the assessment, the nurse observes there is a marked decrease in the client's blood pressure. Which intervention should the nurse implement to help alleviate this client's condition? Place the client in the left lateral position. Place the client in an orthopneic position. Keep the client's legs slightly elevated. Keep the head of the client's bed slightly elevated.

Place the client in the left lateral position. Explanation: The symptoms experienced by the client indicate supine hypotension syndrome. When the pregnant woman assumes a supine position, the expanding uterus exerts pressure on the inferior vena. The nurse should place the client in the left lateral position to correct this syndrome and optimize cardiac output and uterine perfusion. Elevating the client's legs, placing the client in an orthopneic position, or keeping the head of the bed elevated will not help alleviate the client's condition.

A G3P2 woman at 39 weeks' gestation presents highly agitated, reporting something "came out" when her membranes just ruptured. Which action should the nurse prioritize after noting the umbilical cord is hanging out of the vagina? Contact the health care provider and prepare the client for an emergent vaginal birth. Place the client in Trendelenburg position and gently attempt to reinsert the cord. Put the client in bed immediately, call for help, and lift the presenting part of the fetus off the cord. With the client in lithotomy position, hold her legs and sharply flex them toward her shoulders.

Put the client in bed immediately, call for help, and lift the presenting part of the fetus off the cord. Explanation: The nurse must put the woman in a bed immediately, while calling for help, and holding the presenting part of the fetus off the cord to ensure its safety. Umbilical cord prolapse occurs when the umbilical cord slips down in front of the presenting part, which can result in the presenting part compressing the cord, cutting off oxygen and nutrients to the baby, and the baby is at risk of death. This is an emergency. When a prolapsed cord is evident the nurse does not put the woman in lithotomy position, and cannot attempt to reinsert the cord. A vaginal birth is contraindicated in this situation.

A nurse is caring for a client who is 3 days postpartum and is attempting to breastfeed. Which of the following findings indicate mastitis? Swelling in both breasts Cracked and bleeding nipples Red and painful area in one breast A white patch on a nipple

Red and painful area in one breast Mastitis often appears as a red, hard, and painful area on the breast, commonly in the upper outer quadrant. Although mastitis can occur in both breasts, it is usually unilateral. A client who has mastitis can also influenza-like manifestations, such as fever, chills, headache, and myalgia. After delivery, the nurse should instruct the client to observe the breasts for indications of mastitis and to notify her provider if they occur.

A client in her 10th week of gestation arrives at the maternity clinic reporting morning sickness. The nurse needs to inform the client about the body system adaptations during pregnancy. Which factors correspond to the morning sickness period during pregnancy? Select all that apply. elevated human chorionic gonadotropin (hCG) increased red blood cell (RBC) production elevated human placental lactogen (hPL) increased estrogen level reduced stomach acidity

Reduced stomach acidity Elevated hCG Increased estrogen level The hCG levels in a normal pregnancy usually double every 48 to 72 hours, until they reach a peak at approximately 60 to 70 days after fertilization. This elevation of hCG corresponds to the morning sickness period of approximately 6 to 12 weeks during early pregnancy. Reduced stomach acidity and high levels of circulating estrogens are also believed to cause morning sickness. Elevation of hPL and RBC production do not cause morning sickness. hPL increases during the second half of pregnancy, and it helps in the preparation of mammary glands for lactation and is involved in the process of making glucose available for fetal growth by altering maternal carbohydrate, fat, and protein metabolism. The increase in RBCs is necessary to transport the additional oxygen required during pregnancy.

Symptoms of neonatal abstinence syndrome

WITHDRAWAL - wakefulness - irritability - temp variation; tachycardia; tremors - hyperactivity, high pitched cry, Hyperreflexia, hypertonic -diarrhea, diaphoresis, disorganized suck - respiratory distress, rub marks, rhinorrhea - Apneic attacks, autonomic dysfunction - Weight loss or failure to gain weight -alkalosis (respiratory) - lacrimation MAIN- frequent sneezing high-pitched cry frequent yawning FAS- small head, small brain, joint and limb and finger deformities

A G2P1 woman is in labor attempting a VBAC, when she suddenly complains of light-headedness and dizziness. An increase in pulse and decrease in blood pressure is noted as a change from the vital signs obtained 15 minutes prior. The nurse should investigate further for additional signs or symptoms of which complication? Uterine rupture Umbilical cord compression Placenta previa Hypertonic uterus

Uterine rupture Explanation: The client with any prior history of uterus surgery is at increased risk for a uterine rupture. A falling blood pressure and increasing pulse is a sign of hemorrhage, and in this client a uterine rupture needs to be a first consideration. The scenario does not indicate a hypertonic uterus, a placenta previa, or umbilical cord compression.

A woman in labor with a history of drug and alcohol addiction and no prenatal care has arrived in the emergency department in active labor with cervix 5 cm dilated. An ultrasound shows the fetus in a breech presentation. As a nurse prepares to care for this woman, which assessments indicate the fetus is in distress and needs immediate help to survive? Select all that apply. minimal movement as the fetus descends into the pelvic opening variable deceleration FHR pattern noted on monitor fetal heart rate decreasing into the 80s premature rupture of membranes with yellow-green color fluid that smells foul meconium staining noted on fluid seeping from vagina

Variable deceleration FHR pattern noted on monitor Fetal heart rate decreasing into the 80s Explanation: Variable deceleration fetal heart rate pattern noted on the monitor signifies a possible prolapsed cord. Low fetal heart rate can be a sign of hypoxia. Meconium staining occurs because of cervical pressure on the buttocks and rectum, not because of fetal anoxia, so it is not a sign of fetal distress. It is normal for the fetus to slow in movement as it makes its way through the birth canal.

A nurse is assisting with the care of a newborn immediately following birth. Which of the following medications should the nurse anticipate administering? (Select all that apply) Vitamin K injection Hepatitis B immunization Antibiotic ointment to both eyes Lidocaine gel to the umbilical stump Haemophilus influenza type b immunization (Hib)

Vitamin K injection Hepatitis B immunization Antibiotic ointment to both eyes Vitamin K injection is correct. Vitamin K is administered by a single intramuscular injection of 0.5 to 1 mg soon after birth to prevent hemorrhagic disease of the newborn. Vitamin K is not present in the gastrointestinal tract of the newborn, but production will begin at about 7 days of age as bacteria begin to form in the intestines. Hepatitis B immunization is correct. Hepatitis B immunization is recommended at birth, 1 to 2 months, and between 6 to 18 months. It is injected intramuscularly soon after birth. For newborns born to hepatitis-infected mothers, hepatitis B immune globin (HBIG) also should be administered within 12 hr of birth. The vastus lateralis is the preferred site of intramuscular injections in newborns, and no more than 0.5 mL should be administered in one injection. Parental consent must first be obtained prior to the administration of this immunization. Antibiotic ointment to both eyes is correct. Due to the risk of eye infections to newborns born to women who have vaginal infections (e.g., chlamydia, gonorrhea), the instillation of prophylactic antibiotics is mandatory in the United States. The medication used is dependent upon facility protocols but is usually erythromycin or tetracycline ophthalmic ointment. It is administered into both eyes within 1 to 2 hr after birth. It should be applied from inner canthus to outer canthus, being careful not to touch the eye. After 1 min, the excess ointment can be wiped off.

There are several women in active labor on the unit. Which woman is at highest risk for developing hypotonic contractions and therefore will need frequent nursing assessments? a 37-year-old G2P1 woman being induced whose last ultrasound at 36 weeks' gestation showed oligohydramnios a G4P3 client who is having twins and wants to experience a "natural birth" a 17-year-old primipara requesting more pain medication every 15 to 30 minutes (and not receiving it) even though there is an epidural catheter in place that is working effectively a 21-year-old primipara woman who does not have a support person with her and is very anxious

a G4P3 client who is having twins and wants to experience a "natural birth" Explanation: Hypotonic contractions occur during the active phase of labor and tend to occur after the administration of analgesia in a uterus that is overstretched by a multiple gestation or polyhydramnios, or in a uterus that is lax from grand multiparty. Anxiety is not listed as a cause for hypotonic contractions.

A woman is 10 weeks' pregnant and tells the nurse that this pregnancy was unplanned and she has no real family support. The nurse's most therapeutic response would be to: offer to meet with the client on a regular basis to provide her someone to talk to about her concerns. tell her to move home so her family will be nearby to help her. remind her that she is still early in the pregnancy and she will feel better about it as the pregnancy progresses. encourage her to identify someone that she can talk to and share the pregnancy experience.

encourage her to identify someone that she can talk to and share the pregnancy experience. Explanation: A pregnant woman without social support needs to identify someone with whom she can share the experience of pregnancy because social support is a crucial part of adapting to parenthood. Telling her to move home and telling her that she will feel better as the pregnancy progresses do not address the issue of isolation. Also, moving home may not be a possibility for this woman. The nurse should maintain a professional relationship and not commit to a long-term relationship with a client.

The nurse is monitoring the uterine contractions of a woman in labor. The nurse determines the woman is experiencing hypertonic uterine dysfunction based on which contraction finding? well coordinated. poor in quality. brief. erratic.

erratic Explanation: Hypertonic contractions occur when the uterus never fully relaxes between contractions, making the contractions erratic and poorly coordinated because more than one uterine pacemaker is sending signals for contraction. Hypotonic uterine contractions are poor in quality, brief, and lack sufficient intensity to dilate and efface the cervix.

A client at 38 weeks' gestation has an ultrasound performed at a routine office visit and learns that her fetus has not moved out of a breech position. Which intervention does the nurse anticipate for this client? vacuum extraction external cephalic version trial labor forceps birth

external cephalic version Explanation: External cephalic version is the turning of a fetus from a breech to a cephalic position before birth. It may be done as early as 34 to 35 weeks, although the usual time is 37 to 38 weeks of pregnancy. A trial birth is performed when a woman has a borderline (just adequate) inlet measurement and the fetal lie and position are good and involves allowing labor to take its normal course as long as descent of the presenting part and dilation (dilatation) of the cervix continue to occur. Forceps, which are not commonly used anymore, and vacuum extraction are used to facilitate birth when other complications are present, but they would be less likely to be used with a fetus in breech position.

A client at 35 weeks' gestation is now in stable condition after being admitted for vaginal bleeding. Which assessment should the nurse prioritize? fetal heart tones signs of shock infection uterine stabilization

fetal heart tones Explanation: When a client is admitted for vaginal bleeding and is stable, the next priority assessment is to determine if the fetus is viable. The other options are not a higher priority than fetal heart tones.

A client arrives to the clinic very excited and reporting a positive home pregnancy test. The nurse cautions that the home pregnancy test is considered a probable sign and will assess the client for which sign to confirm pregnancy? fetal movement felt by examiner positive office pregnancy test Hegar sign Chadwick sign

fetal movement felt by examiner Explanation: The positive signs of pregnancy are fetal image on a sonogram, hearing a fetal heart rate, and examiner feeling fetal movement. A pregnancy test has 95% accuracy; however, it may come back as a false positive. Hegar sign is a softening of the uterine isthmus. Chadwick sign may have other causes besides pregnancy.

A client has arrived to the birthing center in labor, requesting a VBAC. After reading the client's previous history, the nurse anticipates that the client would be a good candidate based on which finding? had prior transfundal uterine surgery had previous lower abdominal incision had prior classic uterine incision has a contracted pelvis

had previous lower abdominal incision Explanation: The choice of a vaginal or repeat cesarean birth can be offered to women who have had a lower abdominal incision. Contraindications to VBAC include a prior classic uterine incision, prior transfundal uterine surgery, uterine scar other than low-transverse ("bikini cut") cesarean scar, contracted pelvis, and inadequate staff at the facility if an emergency cesarean birth is required.

At 31 weeks' gestation, a 37-year-old client with a history of preterm birth reports cramps, vaginal pain, and low, dull backache accompanied by vaginal discharge and bleeding. Assessment reveals cervix 2.1 cm long; fetal fibronectin in cervical secretions, and cervix dilated 3 to 4 cm. Which interactions should the nurse prepare to assist with? hospitalization, tocolytic, and corticosteroids bed rest and hydration at home an emergency cesarean birth careful monitoring of fetal movement (kick) counts

hospitalization, tocolytic, and corticosteroids Explanation: At 31 weeks' gestation, the goal would be to maintain the pregnancy as long as possible if the client and fetus are tolerating the continuation of the pregnancy. Stopping the contractions and placing the client in the hospital allows for monitoring in a safe place if the client continues and gives birth. Administration of corticosteroids may help to develop the lungs and prepare for early preterm birth. Sending the client home is contraindicated in the scenario described. An emergency cesarean birth is not indicated at this time. Monitoring fetal movement (kick) counts is typically done with a postterm pregnancy.

A nurse is caring for a newborn who has macrosomia and whose mother has diabetes mellitus. The nurse should recognize which of the following newborn complications as the priority focus of care? hypoglycemia hypomagnesemia hyperbilirubinemia hypocalcemia

hypoglycemia Newborns of mothers who have diabetes are at high risk for hypoglycemia due to the loss of high levels of glucose after the umbilical cord is cut. This results in fetal hyperinsulinemia. It can take several days for the newborn to adjust to secreting appropriate amounts of insulin for the lower level of blood glucose. Because severe hypoglycemia can lead to cyanosis and seizures, prevention of hypoglycemia becomes the nurse's priority focus of care.

hypotonic labor patterns

hypotonic contractions -Irregular contractions -Lack intensity -Less than 1 cm dilatation per hour usually caused by Cephalopelvic Disproportion (CPD) or Malposition

A pregnant woman has just found out that she is having twin girls. She asks the nurse the difference between fraternal and identical twins. The nurse explains that with one set of twins there is fertilization of two ova, and with the other set one fertilized ovum splits. What type of twins result from the split ovum? identical both types can result from the split ovum neither type results from a split ovum fraternal

identical Explanation: The incidence of twins is about 1 in 30 conceptions, with about 2/3 being from the fertilization of two ova (fraternal) and about 1/3 from the splitting of one fertilized ovum (identical).

how does hemolytic or incompatibility jaundice occur

if mom is Rh negative or type O

A 28-year-old primigravida client with diabetes mellitus, in her first trimester, comes to the health care clinic for a routine visit. The client reports frequent episodes of sweating, giddiness, and confusion. What should the nurse tell the client about these experiences? use of insulin needs to be reduced as pregnancy advances insulin resistance becomes minimal in the latter half of the pregnancy tissue sensitivity to insulin increases as pregnancy advances increased secretion of insulin occurs in the first trimester

increased secretion of insulin occurs in the first trimester Explanation: Increased secretion of insulin in the maternal body in the first trimester is due to the rise in serum levels of estrogen, progesterone, and other hormones. During the second half of pregnancy, tissue sensitivity to insulin progressively decreases, producing hyperglycemia and hyperinsulinemia. Use of insulin needs to be increased not reduced as pregnancy advances. Insulin resistance becomes maximal not minimal in the latter half of the pregnancy.

A nursing student working with a client in preterm labor correctly identifies which medication as being used to relax the smooth muscles of the uterus and for seizure prophylaxis and treatment in clients with preeclampsia? betamethasone nifedipine magnesium sulfate indomethacin

magnesium sulfate Explanation: The drug used to relax the uterine muscles and for seizure prophylaxis is magnesium sulfate. Betamethasone promotes fetal lung maturity, indomethacin inhibits uterine activity to arrest preterm labor, and nifedipine blocks calcium movement into the muscle cells and inhibits preterm labor.

The nurse advises a pregnant client to keep a small high-carbohydrate, low-fat snack at the bedside. The nurse should point out this will assist with which condition? faintness slowed GI transit time nausea and vomiting heartburn

nausea and vomiting Explanation: Women will commonly experience nausea and vomiting upon awakening first thing in the morning. Clients who experience this should be encouraged to have small snacks at their bedside for eating prior to moving from the bed. Heartburn is a result of pressure and hormone action. Faintness is due to pressure on the vena cava, not blood sugar. GI transit time is not affected.

The nurse is advising a pregnant woman during her first prenatal visit regarding the frequency of future visits. Which schedule is recommended for prenatal care? once every 4 weeks for the first 36 weeks, then weekly until the birth once every 4 weeks for the first 28 weeks, then every 3 weeks until 36 weeks, and then every 2 weeks until the birth once every 4 weeks for the first 28 weeks, then every 2 weeks until 36 weeks, and then weekly until the birth once every 3 weeks for the first 28 weeks, then every 2 weeks until 36 weeks, and then weekly until the birth

once every 4 weeks for the first 28 weeks, then every 2 weeks until 36 weeks, and then weekly until the birth Explanation: The best health for mother and baby results when the mother has her first visit before the end of the first trimester (before the end of week 13) and then has regular visits until after she has delivered the baby. The usual timing for visits is about once every 4 weeks for the first 28 weeks, then every 2 weeks until 36 weeks, and then weekly until the birth.

When caring for a client requiring a forceps-assisted birth, the nurse would be alert for: increased risk for uterine rupture. damage to the maternal tissues. potential lacerations and bleeding. increased risk for cord entanglement.

potential lacerations and bleeding. Explanation: Forcible rotation of the forceps can cause potential lacerations and bleeding. Cervical ripening increases the risk for uterine rupture in a client attempting vaginal birth after undergoing at least one previous cesarean birth. There is an increased risk for cord entanglement in multiple pregnancies. Damage to the maternal tissues happens if the cup slips off the fetal head and the suction is not released.

The nursing student doing a rotation in obstetrics is talking to her preceptor about dystocia. She asks what is meant by the term "expulsive forces," better known as the "powers." The preceptor correctly tells her that the "powers" include which factors? Select all that apply. fetal development analgesia mother's age position presentation

presentation position fetal development Explanation: Dystocia can result from problems or abnormalities involving the expulsive forces (known as the "powers"): presentation, position, and fetal development. The others are not included in the "powers."

A client's menstrual period is two weeks late. She has been feeling tired and has had episodes of nausea in the morning. What classification of pregnancy symptoms is this client experiencing? no classification probable presumptive positive

presumptive Explanation: The most common presumptive sign of pregnancy is a missed menstrual period, or amenorrhea. Other presumptive signs include nausea, fatigue, swollen, tender breasts, and frequent urination.

The nurse is assessing a pregnant woman and noticing behavior changes that indicate she is beginning to accomplish the maternal tasks of becoming a mother. The client is in her third trimester. Which behavior would the nurse most likely assess? identifying what must be given up to assume her new role questioning of her ability to become a good mother acknowledging the fetus as a separate entity accepting the pregnancy but not yet the fetus

questioning of her ability to become a good mother Explanation: During the third trimester, the woman often questions her ability to become a mother. Accepting the pregnancy but not the fetus and identifying what must be given up to assume new roles are characteristic of the first trimester. Learning how to delay own desires and acknowledging the fetus as a separate entity are characteristic of the second trimester.

A 19-year-old nulliparous woman is in early labor with erratic contractions. An assessment notes that she is remaining at 3 cm. There is also a concern that the uterus is not fully relaxing between contractions. The nurse suspects which complication? cephalopelvic disproportion precipitate labor reduced oxygen to the fetus ruptured uterus

reduced oxygen to the fetus Explanation: Hypertonic uterine dysfunction occurs when the uterus never fully relaxes between contractions. Placental perfusion becomes compromised, thereby reducing oxygen to the fetus. This occurs in early labor and affects nulliparous women more than multiparous women. A ruptured uterus is a potential complication; however, hypoxia to the fetus would occur first. Cephalopelvic disproportion is usually associated with hypotonic uterine dysfunction. Precipitate labor is one that is completed in less than 3 hours from the start of contractions to birth.

A client who has just given a blood sample for pregnancy testing in the health care provider's office asks the nurse what method of confirming pregnancy is the most accurate. The nurse explains the difference between presumptive symptoms, probable signs, and positive signs. What should the nurse mention as an example of a positive sign, which may be used to diagnose pregnancy? laboratory test of a urine specimen for hCG absence of a period visualization of the fetus by ultrasound laboratory test of a blood serum specimen for hCG

visualization of the fetus by ultrasound Explanation: There are only three documented or positive signs of pregnancy: 1) demonstration of a fetal heart separate from the mother's, 2) fetal movements felt by an examiner, and 3) visualization of the fetus by ultrasound. The absence of a period is an example of a presumptive symptom, which is a symptom that, when taken as a single entity, could easily indicate other conditions. Laboratory tests of either urine or blood serum for human chorionic gonadotropin (hCG) are examples of probable signs of pregnancy, which are objective and so can be verified by an examiner.

Prolapsed Cord

when a fetus's umbilical cord slips through the cervix and into the vagina after a mother's water breaks and before the baby descends into the birth canal


Ensembles d'études connexes

Adv. Bio 1 Keystone Review Packet Questions, Vocab, and Notes

View Set

Chapter 5: Big Data & Marketing Analytics

View Set

PrQ31: Practice Quiz - Ch. 31: Inflation and the Quantity Theory of Money

View Set

JB Practice Test Bank OB & Pediatrics

View Set

Financing Residential Real Estate

View Set

// FAD DIETS VOCAB AND QUESTIONS

View Set

(Chapter 7: Distribution of Sample Means)

View Set